Vous êtes sur la page 1sur 68

MEDICINE

MAY JUNE 2007


PAPER 1

EMQ 1 -Abdominal pain

Abdominal aneurysm- can present as


1. Asymstomatic- can be an incidental finding, screenin gwith men aged 65-75 reduces the mortality,
selective screening can be cost effective those with a family history, known peripheral vascular disease
age 60-80, family history of aaa, know extremity aneurysm.

2.Symptomatic- pulsatile, expansile mass may cause pain in the central abdomen, back , loin and iliac
fossa. Thrombus within the sac may be a soucre of emboli to the lower limbs...trash foot. The aneurysm
may compress surrounding structures such as the IVC, duodenum, and IVC.

3, Rupture- usually in the retroperitoneum , patient have, severe abdominal pain, pulsatile abdominal
mass, present with features of shock.

Appendicitis
Classically poorly localized periumbilical pain which moves to the right iliac fossa and becomes sharper
and constant, anorexia, vomiting is rarely prominent. diarrhoea and constipation may occur. remember
diagnosis is a clinical one, cbc may show inc wbc, and an abd uss can show swollen non compressible
mass, with free fluid.

Constipation
bowel habit varies with each person according to diet. An accepted definition is infrequent passage of
stool < 3 times weekly, with difficulty, straining or discomfort. Alot of causes(ohcm pg 241), i will list
some though, poor diet, inadequate hydration, immoblity, IBS, old age, post op, anal fissure, anal
stricture, colorectal ca, hypercalcemia, hypothyroidsm.

COLON CANCER
Features may be related to the site of the tumor

RIGHT SIDED: present later on when tumor has been around for a while

LEFT SIDED: present more acutely as the tumors are annular (constricting) and stool is more solid

1
Feature RIGHT SIDED LEFT SIDED RECTAL

Common presentation Abdominal mass Obstruction

PR bleed Bleeding- can be occult Can present with dark Bright red-
red bleeding coating stool
Pain Abdominal pain Colicky pain due to Tenesmus
obstruction
Weight loss Common Uncommon

Bowel habits Constipation Constipation & spurious Pencil stool


diarrhoea

Examination:

Anemia, Weight loss, Cachexia, Abdominal mass – tumor or hepatic mets, Abdominal distension –
obstruction or ascites, Rectal mass or blood on glove

Peptic ulcer disease


Epigastric pain, heart burn, bloating fullness after eating meals, may have reflux. duodenal ulcers
commoner, pain before meals or at night, relieved by eating or drinking milk. gastric ulcers, pain after
meals relieved by antacids, + or - weight loss.

ALARM symptoms- Anaemia, loss of weight, anorexia, recent onset of progressive symptoms, melaena or
haematemesis.

Kidney stone

Renal calculi
Remember the types of stones and the incidence??? , the patient may be asymptomatic or present with
a variety of symptoms. Stones in the kidney can cause loin pain. ureteric stones cause colicky pain, loin
to groin. bladder or urethral sontes cause pain on micturition , strangury or interruption of urine flow.

Ruptured ectopic Pregnancy


Lower abdominal pain worse on the affected side, later becoming generalized, pain worse on movement,
may have shoulder tip pain, period of amenorrhoea, vaginal bleeding which tends to follow is usually
mild, dizziness, weakness, fainting,. the patient may have signs of shock, abd exam may reveal
suprapubic distension. tenderness, guarding, rebound tenderness are almost always present. on
bimanual exam the cervix is soft, marked cervical exicitation, adnexa are tender and a mass is
uncommon.

2
Endomertriosis
dull pain in mid pelvis or lower abdomen, dysmenorrhoea, menorrhagia, dyspareunia, infertility,
constipation can occur when it involves the rectosigmoid area, bladder involvement can cause cyclical
hematuria.

Pyelonephritis
infection of the renal pelvis, high fever, rigors, vomiting, loin pain and tenderness, oliguria if in renal
failure, foul smelling urine.

treatment cefuroxime 1.5g/24hr iv then oral 7 day course

Perfoated peptic ulcer


acute onset of severe unremitting epigastric pain, patient is pale shocked, peripherally shut down,
shoulder tip pain from peritonitis,vomiting may occur, the abdomen does not move freely with
respiration, marked tenderness, guarding, fear of meovement and board like ridigity. respiration shallow
and bowel sounds absent.

PID
pain in lower abdomen, pyrexia, offensive vaginal discharge, abnormal uterine bleeding, nausea and
vomiting, dyspareunia, dysuria, menorrhagia, dysmenorrhoea. lower abdominal tenderness, guarding
and ridigity, and marked cervical excitation pain on digital examination.

Bowel obstruction
4 cardinal features, pain, vomiting, constipation and distension.

pain is colicky, short intervals sbo long lbo, constant pain is starangulation. vomiting is an early feature of
sbo and late of lbo, constipation early in lbo and late in sbo, distension is more marked in lbo than sbo.

Pancreatitis
gradual or sudden onset of severe epigastric pain or central abdominal pain, radiating to back, vomiting
is prominent, sitting forward may relieve pain. signs of shock, fever, jaundice, ileus, rigid abdomen,
cullen's sign, grey turner's sign.

Diverticulitis
pyrexia, inc in wbc, a tender colon + or - generalized peritonism, left sided appendicitis. there may be a
history of altered bowel habit, left sided colic relieved by dedecation, nausea, and flatulence.

1. k

3
2. c she is hypothyroid
3.d . 4.f 5.m
Theme2: Polyuria

A. Diabetes mellitus
B. Hypercalcemia
C. Hyperuricemia
D. Nephrogenic diabetes insipidus
E. Hypokalemia
F. Cranial diabetes insipidus
G. Polyuria
H. Renal impairment
I. Thyrotoxicosis
J. Hypoadrenalisim

6. a 56 year old obese gentleman sustained head injury resulting in loss of consciousness for 48 hours.
He passes more than 3 litres of urine per 24 hrs and the serum sodium is 156mmol/L Cranial head
trauma with polyuria and hypernatremia = cranial diabetes inspidus ans F

7. R. Ramsawak a 73 year old male with a 10 yr hx of poorly controlled htn noted that he would awake
x5/night to pee. The urine volume was large and the urine clear. He denied hesitancy or poor stream.
Long standing htn may lead to renal impairment-> nocturia. Could also be due to beta blockers or
diuretics. Ans H renal impairment.

8. a 33 yr old non-pregnant female with galactorrhea was recently admitted for an abdominal pain which
radiated from the loin to the groin. The was an associated urgency and painful hematuria but no fever.
She had been experiencing polyuria one year previously. Patient has MEN 1 syndrome: hypercalcemia
causing stones secondary to parthyroid adenoma and glactorrhea secondary to a pituitary adenoma.
Ans B hypercalcemia

9. A 23 year old female on routine evaluation was noted to be hypertensive. She admitted for polyuria of
5 months duration which was associated with paraesthesia and occasional weakness. Patient conn
syndrome E hypokalemia

10. a 60 year old female complained of fatigue, polyuria and weight loss and increased appetite for 2
months with normal urinalysis. She had frequent bowel motions. These symptoms fit both DM and
hyperthyroidism however abnormalities would be detected on dip stick in DM (++ glucose) Ans I
Thyrotoxicosis

Hypercalcemia: Signs & symptoms


Bones, stones, groans, and psychic moans. Abdominal pain; vomiting; constipation; polyuria; polydipsia;
depression; anorexia; weight loss; tiredness; weakness; hypertension, confusion; pyrexia; renal stones;
renal failure; corneal calcification; cardiac arrest. ECG: Q-T interval.

Causes:

4
Most commonly malignancy (myeloma, bone metastases) and 1° hyperparathyroidism. Others include
sarcoidosis, vit D intoxication, and familial benign hypocalciuric hypercalcaemia (rare; defect in calcium-
sensing receptor). Pointers to malignancy are: increasing albumin, decreasing Cl-, decreasing K+,
alkalosis, increasing PO and alk phos. Other investigations (eg isotope bone scan, CXR, FBC) may also be
of diagnostic value.

MEN type-1 (=MEN1):

Parathyroid hyperplasia/adenoma (~95%; most hypercalcemia).

Pancreatic endocrine tumours (~70%) usually gastrinoma or insulinoma, rarely, somatostatinoma, 1


glucagonomas or VIPoma .

Pituitary adenoma (~50%)usually prolactinoma or GH secreting tumour (acromegaly: p218).

Also, adrenal and carcinoid tumours are associated.

The MEN1 gene is a tumour suppressor gene. Menin, its protein, alters transcription activation. 66

Many are sporadic, presenting in the 3rd-5th decades.

Hyperuricemia: Causes
 High levels of urate in the blood (hyperuricaemia) may result from increased turnover or
reduced excretion of urate. Either may be drug-induced.
 Drugs: Cytotoxics; thiazides; pyrazinamide.
 Increased cell turnover: Lymphoma; leukaemia; psoriasis; haemolysis; muscle death
(rhabdomyolysis, Tumour lysis syndrome)
 Reduced excretion: Primary gout (p534); chronic renal failure; lead nephropathy;
hyperparathyroidism; pre-eclampsia (OHCS p48).
 In addition: Hyperuricaemia may be associated with hypertension and hyperlipidaemia. Urate
may be raised in disorders of purine synthesis such as the Lesch-Nyhan syndrome.

Hyperuricaemia and renal failure

Severe renal failure from any cause may be associated with hyperuricaemia, and very rarely this may give
rise to gout. Sometimes the relationship of cause and effect is reversed so that it is the hyperuricaemia
that causes the renal failure. This can occur following cytotoxic treatment (tumour lysis syndrome), eg in
leukaemia; and in muscle necrosis.

Diabetes Insipidus:
Defective regulation of water balance secondary to decreased pituitary secretion of vasopressin (central
diabetes insipidus [DI]) or failure of response to vasopressin (nephrogenic DI)

Pathophysiology

 Central DI:

5
o Inadequate secretion of vasopressin may be due to loss or malfunction of the
neurosecretory neurons that make up the neurohypophysis (posterior pituitary) and the
pituitary stalk.

 Nephrogenic DI:

o Insensitivity to vasopressin

o A disorder of renal tubular function resulting in inability to respond to vasopressin in


absorption of water

Etiology

 Central DI:

o Inadequate secretion of vasopressin owing to pathologic condition that may be


idiopathic or familial

o Trauma

o Neurosurgery

o Tumors (e.g., craniopharyngioma, lymphoma, metastasis)

o Idiopathic

o Infections (e.g., meningitis, encephalitis)

o Granulomas (e.g., sarcoid, histiocytosis)

o Vascular disorders

 Nephrogenic DI (insensitivity to vasopressin): Genetic defect in resorption of water in renal


tubule (collecting ducts)

History

 Thirst/polydipsia (with a particular preference for cold or iced drinks)

 Polyuria

 Nocturia

 Dehydration

 Headache

 Visual disturbance

 Rate of onset of polydipsia is more rapid in central DI versus nephrogenic DI.

 Family history of polyuria

Lab findings:

6
In assessing patients with suspected diabetes insipidus (DI), the urine specific gravity of the first morning
urine is helpful in assessing renal ability to concentrate urine. Dilute urine with a relatively high serum
sodium and osmolarity effectively establishes the diagnosis. The serum sodium may be as high as 170
mEq/L, while the serum osmolarity is greater than 300 mOsm/kg. Patients with prerenal azotemia
present with severe dehydration.

Nocturia and renal impairement:


Urine volume or flow rate is determined by solute load, osmolality of medullary interstitium, and
response to vasopressin. In the normal circumstance, sodium, potassium, and chloride excretion are high
during the day and low at night. Plasma vasopressin levels are higher at night than in the day. Thus we
excrete most of our solute load, especially electrolytes, during the waking hours and elaborate a
concentrated acid urine of low solute content overnight. Loss of the diurnal variation of solute excretion
or the ability to raise the urine osmolality above plasma will result in increased nocturnal urine flow
without an obvious increase in daytime urine volume. Even with isosthenuria, the normal daily solute
load of 450 to 750 mOsm will be excreted in 1500 to 2500 ml of urine. In contrast, the absence of
vasopressin effect will result in persistent hyposthenuria (water diuresis) and cause daytime polyuria as
well as nocturia. Similarly, a substantial increase in solute excretion will cause daytime polyuria with
nocturia.

The nocturia of chronic renal disease has been traditionally attributed to the early loss of concentrating
ability that accompanies most renal disease regardless of etiology. Careful analysis of excretory patterns
of patients with chronic renal disease reveals the principal cause of the nocturia to be increased
nocturnal solute excretion. A similar nocturnal saluresis in some renal transplant recipients accounts for
the nocturia that may persist for up to a year after transplantation.

The administration of beta-adrenergic blocking drugs reverses the normal day–night pattern of sodium
excretion. The effect is most pronounced in people with incipient or overt heart failure, but has a lesser
effect in normal individuals as well. It may result in nocturia. Diuretics with a long duration of action
(chlorthalidone) or potent short-acting diuretics (furosemide) taken shortly before bedtime will effect a
nocturnal sodium diuresis.

Hypertension and diabetes type 1:

Hypertension occurs more frequently in persons with type I diabetes, or insulin-dependent diabetes
mellitus (IDDM), than in those with type II diabetes after adjustment for age. Diabetes mellitus of
childhood onset is usually type I, and hypertension occurs with greater frequency in such children.76
Hypertension in children or adolescents with type I diabetes often reflects incipient nephropathy but
may represent essential hypertension as well.

Hyperthyroidisim
History

 Thyrotoxicosis is a hypermetabolic state where energy production exceeds the body needs,
causing increased heat production, diaphoresis, and even fever.

 Thyrotoxicosis affects several different systems:

7
o Constitutional: Fatigue, weakness, increased appetite, weight loss

o Neuropsychiatric: Agitation, anxiety, emotional lability, psychosis, coma, poor


concentration, and memory

o GI: Increased appetite, hyperdefecation

o Gynecologic: Oligomenorrhea, amenorrhea.

o Cardiovascular: Increased heart rate (most common manifestation) and chest discomfort
that mimics angina

o Polyuria secondary to increased aquaporin expression

Hypokalemia: Clinical findings


 Patients with hypokalemia often have no symptoms, especially if the hypokalemia is mild (serum
potassium 3.0–3.5 mEq/L).

 Neuromuscular (most prominent manifestations):

o Skeletal muscle weakness (proximal > distal muscles, lower limbs > upper) may range
from mild weakness to total paralysis, including respiratory muscles; may lead to
rhabdomyolysis and/or respiratory arrest in severe cases.

o Smooth-muscle involvement may lead to GI hypomotility, producing ileus and


constipation (1)[B].

 Cardiovascular:

o Ventricular arrhythmias; higher risk if underlying CHF, left ventricular failure (LVF),
cardiac ischemia

o Hypotension

o Cardiac arrest

 Renal: Polyuria, polydipsia, nocturia owing to impaired concentrating ability, myoglobinuria

 Metabolic: Hyperglycemia

Hypoadrenailisim:
Adrenocorticotropic hormone (ACTH) deficiency results in diminished cortisol secretion (see
Adrenocortical Hypofunction). Symptoms may include weakness, fatigue, weight loss, and hypotension.
Patients with partial ACTH deficiency continue to have some cortisol secretion and may not have
symptoms until stressed by illness or surgery. Adrenal mineralocorticoid secretion continues, so

8
manifestations of adrenal insufficiency in hypopituitarism are usually less striking than in bilateral
adrenal gland destruction (Addison disease); hyponatremia may occur, especially when ACTH and TSH
deficiencies are both present

EMQ 3 HEART FAILURE (HF)

A. Acute myocarditis
B. Aortic stenosis
C. Congestive cardiomyopathy
D. Endocarditis
E. Endomyocardial fibrosis
F. Hypertrophic cardiomyopathy
G. Ischaemic cardiomyopathy
H. Pancarditis
I. Pericarditis
J. Pulmonary hypertension

11) 28 yr old man presents to gp with sore throat. Reassured and no specific tx given. He returns 3
weeks later with SOB and pain in his joints. O/E he has features of HF, soft systolic and diastolic
murmurs and tender bruise-like nodules on his shins. A throat swab reveals strep viridians. D
12) A 39 yr old woman is seen in A&e after collapsing in town. She recalls rushing for the bus before
feeling ‘faint’. She is concerned because she has had similar symptoms several times before
when exerting herself and her brother recently died suddenly of some hear problem. O/E she
has a thrusting cardiac impulse, midsystolic mumur at left sterna edge, ‘jerky’ pulse and basal
inspiratory crackles. F
13) A 64 yr old man ith DM2 is admitted early hours of a.m with severe SOB. He has been unable to
sleep for the last week because of his symptoms, but feels better in a chari by the window. He
has a 7/12 hx of increasing SOB on exertion. He has been admitted on 3 previous occasions with
AMIs. G
14) A 43 yr old woman admitted with a stroke. O/E she has a dense right hemiplegia, sinus rhythm
but has a pansystolic murmur in the mitral area and signs of LVF. Also has splenomegaly and
microscopic haematuria. D
15) A 30yr old woman with SLE develops sharp central chest pain that is worse on lying back and
somewhat relieved by sitting forward. I

Left Heart Failure Right Heart Failure

Orthopnea RUQ pain (due to hepatic congestion)


Paroxysmal nocturnal dyspnea (PND) Nausea, anorexia,
Fatigue, weight loss Facial engorgement, epistaxis
Dyspnea on exertion (DOE) pulsation in neck and face (TR)
Nocturnal Cough Hepatomegaly
Wheeze Hepatojugular reflex
Nocturia Jugular venous distention (JVD)
S3 gallop, Tachycardia Ascites

9
Diaphoresis ,cold peripheries, Cyanosis
Rales Peripheral edema
Muscle wasting.
MYOCARDITIS
DEFINITION; Inflammation of the myocardium

ETIOLOGY; Viral—coxsackie A or B, echovirus, HIV, CMV, influenza, EBV, hepatitis B virus (HBV),
adenovirus, Bacterial—group A beta-hemolytic strep (rheumatic fever), Corynebacterium,
Meningococcus, B. burgdorferi (Lyme), Mycoplasma pneumonia, Parasitic—Trypanosoma cruzi (Chagas’),
Toxoplasma, Trichinella, Echinococcus, Systemic disease—Kawasaki’s, SLE, sarcoidosis, inflammatory
conditions, Drug allergies—sulfonamides, penicillins, Cocaine, Idiopathic—common

SIGNS AND SYMPTOMS

 Retrosternal or precordial chest pain, Fever, fatigue


 Preceding upper respiratory infection (URI)
 Palpitations, tachycardia, syncope
 Signs of CHF (dyspnea, rales, peripheral edema, JVD)

DIAGNOSIS

 Auscultation—S3/S4, mitral or tricuspid regurgitation, friction rub (if pericardium involved)


 ECG—ST segment changes, T wave inversion, low voltage, atrial arrhythmias, transient AV block
 CXR—often normal, may see cardiomegaly or pulmonary venous congestion
 Echocardiography—hypokinetic wall movements, dilated ventricles/atria, pericardial effusion
 Labs—leukocytosis, elevated ESR, elevated cardiac enzymes (slower rise and fall than acute MI,
troponin I is most sensitive)
 Myocardial biopsy

TREATMENT

 Primarily supportive
 ACE inhibitors reduce necrosis and inflammation, digoxin should be used cautiously as its effects
may be exaggerated by the inflamed myocardium.
 Address etiology if known/applicable e.g., antivirals, antibiotics, diptheria—antitoxin, IV IgG
 Immunosuppressive agents are contraindicated (steroids, cyclosporine, NSAIDs).

AORTIC STENOSIS
ETIOLOGY; Degenerative calcific disease (idiopathic, older population), Bicuspid aortic valve (most
common congenital valve abnormality) around age 40.

SYMPTOMS

 Usually asymptomatic early in course


 Dyspnea

10
 Angina and syncope: Particularly during exercise—peripheral resistance falls, LV pressure
remains the same due to stenotic valve, CO cannot maintain BP causing syncope, low BP to
coronary arteries causes angina
 Heart failure

DIAGNOSIS/SIGNS

 Slow rising pulse, narrow pulse pr. (feel diminished/delayed carotid upstroke parvus et tardus)
 Heaving, non-displaced apex beat, LV heave, aortic thrill
 ESM (heard at the base, left sternal edge and the aortic area, radiates to the carotids).
Paradoxical splitting of S2 rarely. More commonly quiet A 2-> severe AS, A2 may be inaudible
(calcified valve). Ejection click (pliable valve) or an S 4 (more often with bicuspid valves).
 ECG- p mitrale, LVH, Left anterior hemiblock, poor R wave progression, LBBB, complete AV block
(calcified ring),t wave inversion in I, aVL, and left precordial leads.
 Echocardiography=dx
 Calcification of aortic valve may be seen on CXR.

TREATMENT

 Valve replacement is definitive therapy.


o Persistent symptoms
o Aortic orifice < 0.7 cm2 body surface area
o Gradient > 70 mm Hg
 Valvuloplasty produces only temporary improvement as rate of restenosis is very high.

DILATED CARDIOMYOPATHY
 Left or right ventricular enlargement with loss of contractile function causing CHF,
dysrhythmias, or thrombus formation. Not due to hypertension or ischemic heart disease.
Patients typically have slowly progressive symptoms of CHF, leads to death in
approximately 3 years.
 Associations: alcohol, ^BP, haemochromatosis, viral infection, autoimmune, peri- or
postpartum, thyrotoxicosis, congenital (X-linked). Prevalence: 0.2%.
Presentation:
 Symptoms; Fatigue, angina, dyspnoea, pulmonary oedema, RVF, emboli, AF, VT.
 Signs: ^Pulse, low BP, ^JVP, displaced, diffuse apex, S3 gallop, mitral or tricuspid
regurgitation (MR/TR), pleural effusion, oedema, jaundice, hepatomegaly, ascites.
Tests:
 CXR: cardiomegaly, pulmonary oedema.
 ECG: tachycardia, non-specific T wave changes, poor R wave progression.
 Echo: globally dilated hypokinetic heart, low EF. Also MR, TR, LV mural thrombus.
Management:
 Bed rest, diuretics, digoxin, ACE-inhibitor, anticoagulation. Consider cardiac
transplantation.

ENDOCARDITIS

11
Fever + new murmur = endocarditis until proven otherwise.
Classification
 Acute-50% of all endocarditis, occurs on normal valves, presents with acute heart failure.
 Subacute course- Endocarditis on abnormal valves. Predisposing cardiac lesions: aortic or
mitral valve disease; tricuspid valves in IV drug users; coarctation; patent ductus
arteriosus; VSD; prosthetic valves. Endocarditis on prosthetic valves may be
early(acquired at the time of surgery, poor prognosis) or late(acquired haematogenously).

Causes
 Bacteria:
o Any cause of bacteraemia exposes valves to the risk of bacterial colonization.
Often, no cause is found.
o Strep viridans is the commonest (35-50%).
o Others: enterococci; Staph aureus or epidermidis; diphtheroids and icroaerophilic
streptococci.
o Rarely: HACEK group of Gram -ve bacteria (Haemophilus-Actinobacillus-
Cardiobacterium-Eikenella-Kingella); Coxiella burnetii; Chlamydia.
 Fungi: Candida, Aspergillus, and Histoplasma.
 Other causes: SLE (Libman-Sacks endocarditis); malignancy.
Clinical features;
 Septic signs: Fever, rigors, night sweats, malaise, weight loss, anaemia, splenomegaly,
and clubbing.
 Cardiac lesions: Any new murmur, or a change in the nature of a pre-existing murmur,
should raise the suspicion of endocarditis. Vegetations may cause valve destruction, and
severe regurgitation, or valve obstruction. An aortic root abscess causes prolongation of
the P-R interval, and may lead to complete AV block. LVF is a common cause of death.
 Immune complex deposition: Vasculitis may affect any vessel. Microscopic haematuria
is common; glomerulonephritis and acute renal failure may occur. Roth spots (boat-
shaped retinal haemorrhage with pale centre, splinter haemorrhages (on finger or toe
nails) Osler's nodes (painful pulp infarcts in fingers or toes) and Janeway lesions
(painless palmar or plantar macules) are pathognomonic.
 Embolic phenomena: abscesses in left sided- brain, heart, kidney, spleen, GI tract. In
right-sided endocarditis- pulmonary abscesses.

The modified Duke criteria (definite: 2 major criteria, or 1 major and 3 minor criteria, or 5 minor
criteria; possible: 1 major and 1 minor criteria, or 3 minor criteria)
Major clinical criteria:
o Positive blood culture:
 Typical microorganism for infective endocarditis (viridans strep., S.
aureus, or community acquired enterococcus) from 2 separate blood
cultures, or
 Persistently positive blood culture.

12
o Single positive blood culture for Coxiella burnetii or anti–phase-1 IgG antibody
titer >1:800
o Positive ECHO (transesophageal ECHO for prosthetic valves or complicated IE):

 Vegitation, Periannular abscess, New partial dehiscence of prosthetic valve


or New valvular regurgitation (change in preexisting murmur not
sufficient)
 Minor criteria:
o Predisposing heart condition or IV drug use

o Fever ≥38.0°C (100.4°F)

o Vascular phenomena: Major arterial emboli, septic pulmonary infarcts, mycotic


aneurysm, intracranial hemorrhage, conjunctival hemorrhage, Janeway lesions
o Immunologic phenomena: Glomerulonephritis, Osler nodes, Roth spots,
rheumatoid factor
o Microbiologic evidence: Positive blood culture, but not of major criterion
(excluding single positive cultures for coagulase-negative staphylococci and
organisms that do not cause endocarditis) or serologic evidence of infection with
an organism likely to cause infective endocarditis

Blood cx; Take 3 sets at different times and from different sites at peak fever.
Normochromic, normocytic anaemia, neutrophil leucocytosis, high ESR/CRP. check U&E,
Mg2+, LFT.
Urinalysis for microscopic haematuria. CXR (cardiomegaly) and ECG (prolonged P-R interval)
at regular intervals.
Echocardiography; TTE may show vegetations, but only if >2mm. TOE is more sensitive, and
better for visualizing mitral lesions and possible development of aortic root abscess.

Management;
 Antibiotics
 Consider surgery if: heart failure, valvular obstruction; repeated emboli; fungal
endocarditis; persistent bacteraemia; myocardial abscess; unstable infected prosthetic
valve.

Prognosis; 30% mortality with staphylococci; 14% with bowel organisms; 6% with sensitive
streptococci.

HOCM , Hypertrophic cardiomyopathy


HOCM= LV outflow tract (LVOT) obstruction from asymmetric septal hypertrophy.
Prevalence: 0.2%. Autosomal dominant inheritance, but 50% are sporadic. 70% have mutations
in genes encoding beta-myosin, alpha-tropomyosin, and troponin T. May present at any age. Ask
about family history or sudden death.

13
Symptoms & signs:
 Angina; dyspnoea; palpitation; syncope; sudden death (VF is amenable to implantable
defibrillators). Jerky pulse; a wave in JVP; double apex beat; systolic thrill at lower left
sternal edge; harsh ejection systolic murmur.

Tests:
 ECG: LVH; progressive T wave inversion; deep Q waves (inferior + lateral leads); AF;
WPW syndrome; ventricular ectopics; VT.
 Echo: asymmetrical septal hypertrophy; small LV cavity with hypercontractile posterior
wall; midsystolic closure of aortic valve; systolic anterior movement of mitral valve.
 Cardiac catheterization may provoke VT. It helps assess: severity of gradient; coronary
artery disease or mitral regurgitation.
 Electrophysiological studies may be needed (eg if WPW) Exercise test, Holter monitor to
risk stratify.

Management:
 Beta-blockers or verapamil for symptoms Amiodarone for arrhythmias (AF, VT).
 Anticoagulate for paroxysmal AF or systemic emboli. Dual-chamber pacing (is used
if symptomatic despite drugs.
 Septal myomectomy (surgical, or chemical, with alcohol, to lower LV outflow tract
gradient) is reserved for those with severe symptoms. Consider implantable
defibrillator.
Mortality: 5.9%/yr if <14yrs; 2.5%/yr if >14yrs. Poor prognostic factors: age <14yrs or syncope
at presentation; family history of HOCM/sudden death.

ACUTE PERICARDITIS

Inflammation of the pericardium which may be primary or secondary to systemic disease.

Causes:
 Viruses (coxsackie, ‘flu, Epstein-Barr, mumps, varicella, HIV)
 Bacteria (pneumonia, rheumatic fever, TB)
 Fungi, Myocardial infarction, Dressler's
 Others: uraemia, Rheumatoid arthritis, SLE, myxoedema, trauma, surgery, malignancy,
radiotherapy, procainamide, hydralazine.

Clinical features:
 Prodrome of fever, malaise, myalgias
 Acute, sharp, stabbing chest pain, Duration typically hours to days

 Pleuritic pain
 Pain improved by leaning forward, worsened by lying supine

Tests:

14
 ECG classically shows concave (saddle-shaped) ST segment elevation, but may be
normal or non-specific (10%).
 Blood tests: FBC, ESR, U&E, cardiac enzymes (NB: troponin may be raised), viral
serology, blood cultures, Also; autoantibodies, fungal precipitins, thyroid function tests.
 Cardiomegaly on CXR may indicate a pericardial effusion.
 Echo (if suspected pericardial effusion).
Treatment:
Analgesia, eg ibuprofen 400mg/8h PO with food. Treat the cause. Consider colchicine before
steroids/immunosuppressants if relapse or continuing symptoms occur. 15-40% do recur

EMQ #4 hematological malignancies

Leukemia: a type of CA of blood/ bone marrow characterized by an abnormal increase of wbc.

ALL: peak at 2-5 yrs of age, second peak in old age. It is a malig of immature lymphocytes that
replicate to overcrowd the b.marrow decreased production of other blood cells. Can also
infiltrate extra marrow sites: l.nodes, spleen, liver...symptoms related to the pancytopenia that
occurs. Dx confirmed by b.marrow biopsy. Assoc wt Phyladelphia chrom.

CLL: affects B cells, and as with all other leukemia , there is crowding of the b.marrow. mostly
affects adults....assoc wt swollen l.nodes, spleen n liver..... anemia n infections follow. Dx is by
abnormally high B cell # in blood n b.marrow wt atypical cell surface proteins, there is
replication of one B cell ie cloning

AML: most common acute leukemia affecting adults. Symptoms of AML are caused by
replacement of normal bone marrow with leukemic cells, which causes a drop in red blood cells,
platelets, and normal white blood cells. These symptoms include fatigue, shortness of breath,
easy bruising and bleeding, and increased risk of infection. Spleen enlargement can occur but is
mild n asymptomatic...Auer rods are Dx

CML: characterized by the increased and unregulated growth of predominantly myeloid cells
in the bone marrow and the accumulation of these cells in the blood. CML is a clonal bone
marrow stem cell disorder in which proliferation of mature granulocytes (neutrophils,
eosinophils, and basophils) and their precursors is the main finding. Linked to philadelphia
chrom where part of the BCR ("breakpoint cluster region") gene from chromosome 22 is fused
with the ABL gene on chromosome 9.. In pathophys, there is increased cell replication with
decreased DNA repair. Can occur at any age but most commonly in the mid-aged to old pt.
Slenomegally is a common finding

Essential thrombocytopenia: over production of PLT (>2 mths n >600x10^9/l)without a


cause...may myelofibrosis or AML

15
The PLTs do not func properly bleeding and thrombosis...........there may also be
splenomegally

Hodgkins lymphoma: Reed-Sternberg cells

Non-Hodgkin lymphoma: any lymphoma except Hodgkin lymphoma

HODGKIN VS NON-HODGKIN LYMPHOMA

- Both involve specific lymphocytes


- HL: Reed-Sternberg +ve, NHL: Reed-Sternberg –ve
- NHL more common
- Risk of NHL increases wt increasing age
- Both types of lymphoma may also be associated with general symptoms of weight loss,
fevers, and night sweats

Monoclonal gammopathy of uncertain significance: a condition in which a paraprotein is


found in the blood during standard laboratory tests. It resembles multiple myeloma and similar
diseases, but the levels of antibody are lower, the number of plasma cells (white blood cells that
secrete antibodies) in the bone marrow is lower, it has no symptoms or problems, and no
treatment is indicated.

Median age is 70yrs.

 MGUS is characterized by the following:


o Serum M-protein value less than 3 g/dL

o Fewer than 10% plasma cells in the bone marrow

o No or only small amounts of Bence-Jones protein in the urine

o Absence of lytic bone lesions

o No related anemia, hypercalcemia, renal failure, or any related end-organ damage

Multiple myeloma: CA of plasma cells:

2/3 for Dx:

- >10% plasmsa cells in b.marrow aspirate


- Osteolytic bone lesions
- Serum paraprotien or Bence-Jones protien in urine.

Median age of Dx is 62 yrs, low wbc, low plt

Myelofibrosis: replacement of b.marrow by collagenous connective tissue fibres

16
Symptoms: splenomegaly, bone pain, low plteasy bruising, increased susceptability to
infection, anemia

16. ans……….B

17 ans………..D

18 ans………..C

19. ans………F

20. ans………..G

1. Otitis Media
 Most common at 6-12 months of age
 Infants and young children are prone because of a shorter Eustacian tube that is
horizontal and functions poorly
 Pain in the ear, fever, irritability, pulling or tugging of the ear
 Bright red, bulging, loss of normal light reflex (note the light reflex can still sometimes be
seen, but it won’t be normal, it might be decreased or dull)
 RSV, rhinovirus, pneumococcus, H influenza, Moxarella catarrhalis
 Complications: Mastoiditis, Meningitis
 Paracetamol, Ibuprofen, 80% resolve spontaneously
 If unwell after 2-3 days, antibiotics can be given, amoxicillin
 OME: asymptomatic except for decreased hearing
 Eardrum dull and retracted, +/- a fluid level
 1 year olds, resolves spontaneously
 Antibiotics improve TM appearance in short term
 OME = most common cause of conductive hearing loss in kids
 Recurrent = grommets + adenoidectomy

2. Urinary Tract Infection


 UTI occurs usually before the age of 11 and recur within a year
 Can involve pyelonephritis(fever, systemic involvement) or cystitis (no fever or low
grade)
 Important because half have structural abnormalities of the UT, and pyelonephritis can
damage the kidneys by scarring -> HTN, CRF if bilateral
 Presentation varies with age
 Infancy: Vomiting, diarrhea, poor feeding/failure to thrive, Prolonged NNJ, Septicemia,
Febrile convulsion in the >6mths group, Septicaemia
 Childhood:Dysuria, frequency,fever +/- rigors,vomiting, diarrhea, abd/loin pain, febrile
convulsion, recurrence of enuresis

17
3. Meningitis
 Viral infections are the most common cause and are self resolving
 Bacterial infection usually follows bacteremia
 Non specific signs – irritability, poor feeding, vomiting, fever, drowsiness, seizures,
reduced consciousness
 Bulging fontanelle, neck stiffness, opistotonos, photophobia, signs of shock, purpura
 LP, BCx, PCR, throat swab, scrapings of skin, serological dx at 4-6 wks after presentation
 Cefotaxime/ceftriaxone
 Below 3 months give ampicillin with the above
 Dexamethasone after 3 months decreases risk of deafness

4. Acute epiglotitis
 Life threatening emergency due to obstruction
 Caused by H influenza
 Associated with septicemia
 Most common in age 1-6yrs
 Presentation: high fever, toxic looking, intensely painful throat affecting swallowing and
speaking, saliva drools down chin, soft inspiratory stridor, increasing resp difficulty, sits
immobile, upright, open mouth
 Admit, secure airway, blood culture, cefuroxime 3-5 days

5. Pneumonia
 Viruses in young children, bacteria in older children
 Viral fluctuates with seasons, bacterial is throughout the year
 Gp B strep in newborn, Strep pn and RSV in infants
 More than 5 years = Mycoplasma pn, Strep pn
 Fever, difficulty breathing, preceeded by an URTI, unwell child, localized chest,
abdominal, neck pain
 Tachypnea, nasal flaring, chest indrawing, hyperinflation, wheeze, end inspiratory
crackles
 CXR, blood culture, cbc, nasopharyngeal aspirate USS chest

6. Bronchiolitis
 1-9 months, rare after 1 year
 RSV
 Coryzal symptoms, then dry cough, breathlessness, wheeze
18
 P/C – feeding difficulty due to increasing dyspnea, recurrent apnea
 Sharp dry cough, recessions, hyperinflation, fine end inspiratory crackles, high pitch
wheeze expiratory, cyanosis/pallor liver displaced downwards
 Nasopharyngeal aspirates, CXR – hyperinflation of lungs due to small aw obstruction, air
trapping, focal atelectasis

7. Bacterial tracheitis
 Staph aureus or H influenza
 Similar picture to croup except high fever toxic progressively rapid aw obstruction with
thick aw secretions

8. Juvenile rheumatoid arthritis


 Persistent joint swelling, less than 16 years of age, absence of infection or any
identifiable cause
 Systemic onset : spiking fevers, salmon pink rash, anemia, wt loss, hepatosplenomegaly,
arthralgia and myalgia, large and small joints affected, RF negative, TMJ affected
sometimes
 Polyarticular: 5 or more jts affected, symmetrically, large and small, poor wt gain, mild
anemia, morning stiffness RF negative, ANA positive, no eye involvement
 Pauciarticular: up to 4 joints, large joints, knees ankles elbows, minimal systemic
symptoms
 Diagnostic criteria: Age under 16 years, arthritis in one or more jts for m ore than or
equal to 6 weeks, exclude other causes

9. ALL
 Presents over several weeks
 Presents with signs and symptoms of infiltration of bone marrow
 Bone pain, abnormal bruising, lymphadenopathy, hepatosplenomegaly, abnormal
bruising, pallor, infections

10. Bacteremia
 Bacteremia is the presence of viable bacteria in the circulating blood. This may or may
not have any clinical significance because harmless, transient bacteremia may occur
following dental work or other minor medical procedures; however, this bacteremia is
generally clinically benign and self-resolving in children who do not have an underlying
illness or immune deficiency or a turbulent cardiac blood flow. The concern with occult
bacteremia is that it could progress to a more severe local or systemic infection if left
untreated. Most episodes of occult bacteremia spontaneously resolve, and serious

19
sequelae are increasingly uncommon. However, serious bacterial infections occur,
including pneumonia, septic arthritis,osteomyelitis, cellulitis, meningitis, and sepsis.
Patients with occult bacteremia do not have clinical evidence other than fever Fever is
less common in infants younger than 3 months than in those aged 3 months to 3 years.
Young infants may not mount a fever response and may also be hypothermic in response
to illness or stress Of all pediatric patients presenting for evaluation of fever, 20% have
fever for which the source of infection is undetermined after a history and physical
examination.

 Rochester criteria are formal elements of the history that have been widely accepted as
indicating a decreased risk for occult bacteremia in infants aged 60 days or younger.
o Was previously healthy
o Had a term of at least 37 weeks' gestation
o Did not receive perinatal antibiotics
o Was not hospitalized longer than the mother following delivery
o Did not receive treatment for unexplained hyperbilirubinemia
o Not currently using antibiotics
o Has no previous hospitalizations
o Has no chronic or underlying illness
 Elements of the history that indicate an increased risk for occult bacteremia in infants
and children after the neonatal period include the following:
o Age, which determines the cutoff used to define fever
o Febrile temperature (≤ 3 mo and temperature >38°C [100.4°F], 3-36 mo and
temperature ≥ 39-39.5°C [102.2-103.1°F])
o Current antibiotic use
o Previous hospitalizations
o Chronic or underlying illness
o Immunodeficiency (eg, hypogammaglobulinemia, sickle cell anemia, human
immunodeficiency virus [HIV], malnutrition, asplenia)

ANSWERS

21. D ( classic history, see above)

22. C (again, classic history, see above)

23. J (By default this seemed to be the correct-ish answer. I don’t think it can be ALL because they
present more with bone pain than arthritis, and nothing else has a typical joint involvement in their
presentation)

24. G (10 month old with dry cough and intermittent wheeze = bronchiolitis)

25. F ( URTI which progresses…usually it progresses to pneumonia)

20
EMQ 6: SKIN LESIONS

A. ATOPIC ECZEMA

History
Incessant pruritus is the only symptom of atopic dermatitis, children often scratch themselves uncontrollably. Although pruritus may be
present in the first few weeks of life, parents become more aware of the itch as the itch-scratch cycle matures when the patient is aged
approximately 3 months. The disease typically has an intermittent course with flares and remissions occurring, often for unexplained
reasons.
Physical
Primary findings of atopic dermatitis include xerosis, lichenification, and eczematous lesions. Excoriations and crusting are common. The
eczematous changes and its morphology are seen in different locations depending on the age of the patient.

 Infancy
o Atopic dermatitis is usually noticed soon after birth. Xerosis occurs early and often involves the whole body; the diaper
area is usually spared.
o The earliest lesions affect the creases (antecubital and popliteal fossae), with erythema and exudation. Over the
following few weeks, lesions usually localize to the cheeks, the forehead and scalp, and the extensors of the lower legs; however, they
may occur in any location on the body, usually sparing the diaper area. Lesions are ill-defined, erythematous, scaly, and crusted
(eczematous) patches and plaques.
o Lichenification is seldom seen in infancy. A typical presentation is shown in the image below.Typical atopic dermatitis on
the face of an infant.
 Childhood
o Xerosis is often generalized. The skin is flaky and rough.
o Lichenification is characteristic of childhood atopic dermatitis. It signifies repeated rubbing of the skin and is seen mostly
over the folds, bony protuberances, and forehead.
o Lesions are eczematous and exudative. Pallor of the face is common; erythema and scaling occur around the eyes.
Dennie-Morgan folds (ie, increased folds below the eye) are often seen. Flexural creases, particularly the antecubital and popliteal
fossae, and buttock-thigh creases are often affected. See the image below.Flexural involvement in childhood atopic dermatitis.
o Excoriations and crusting are common. The crusting with atopic dermatitis should not be confused with infection
because both may manifest oozing and crusting.
 Adulthood
 Lesions become more diffuse with an underlying background of erythema. The face is commonly involved
B. CANDIDIASIS
and is dry and scaly.
 Xerosis is prominent.
History
 Lichenification may be present.
Candidiasis can cause a wide spectrum of clinical syndromes, as described below. The clinical presentation can vary depending on the
type of infection and the degree of immunosuppression.

Cutaneous candidiasis syndromes

 >Generalized cutaneous candidiasis: This is an unusual form of cutaneous candidiasis that manifests as a diffuse eruption over
the trunk, thorax, and extremities. The patient has a history of generalized pruritus, with increased severity in the genitocrural folds, anal
region, axillae, hands, and feet. Physical examination reveals a widespread rash that begins as individual vesicles that spread into large
confluent areas.
 >Intertrigo: The patient has a history of intertrigo affecting any site in which skin surfaces are in close proximity, providing a warm
and moist environment. A pruritic red rash develops. Physical examination reveals a rash that begins with vesiculopustules that enlarge
and rupture, causing maceration and fissuring. The area involved has a scalloped border with a white rim consisting of necrotic epidermis
that surrounds the erythematous macerated base. Satellite lesions are commonly found and may coalesce and extend into larger lesions
(see image below).Erythema, maceration, and satellite pustules in the axilla, accompanied by soreness and pruritus, result in a form of
intertrigo. Courtesy of Matthew C. Lambiase, DO.
 C. IMPETIGO
Physical >Metastatic skin lesions: Characteristic skin lesions occur in approximately 10% of patients with disseminated candidiasis and
candidemia. The lesions may be numerous or few and are generally described as erythematous, firm, nontender macronodular lesions
Bullous impetigo
with discrete physical
borders. findings
Biopsy are as follows:
specimens of these lesions demonstrate yeast cells, hyphae, or pseudohyphae, and cultures are positive
 for Candidaspecies in approximately 50% of cases.
Bullous impetigo affects neonates most frequently, but it also occurs in older children and adults.[10]
 >Candida folliculitis: The infection is found predominantly in the hair follicles and, rarely, can become extensive.
The characteristic lesion is a vesicle that develops into a superficial flaccid bulla less than 1 cm in diameter on intact skin, with
 minimal or>Paronychia and onychomycosis:
no surrounding redness. Initially,Paronychia
the vesicle and onychomycosis
contains are becomes
clear fluid that frequentlyturbid.
associated with immersion of the hands in
 water and with diabetes mellitus. The patient has a history of a painful and erythematous area around and underneath the nail and nail
The roof of the bulla ruptures, often leaving a peripheral collarette of scale or a tubelike rim at the periphery. A varnishlike crust
bed. Physical examination reveals an area of inflammation that becomes warm, glistening, tense, and erythematous and may extend
develops centrally,
extensively under thewhich,
nail. ifIt removed, reveals
is associated withasecondary
moist red base.
nail thickening, ridging, discoloration, and occasional nail loss.

Intact bullae are not usually present because they are very fragile.

When present, intact bullae do not demonstrate a positive Nikolsky sign.

Lesions of a primary skin disease, such as atopic dermatitis or varicella, may be present.

Lesions may be localized or widely scattered.
 21
Lesions are often found on intertriginous areas such as neck, axillary and crural folds, as well as in the diaper area, but they may
appear on the face or anywhere on the body.

No regional lymphadenopathy is present. In infants, extensive lesions may be associated with systemic symptoms such as fever,
malaise, generalized weakness, and diarrhea. Rarely, infants may present with signs of pneumonia, septic arthritis, or osteomyelitis.

Bullous impetigo is considered to be less contagious than nonbullous impetigo
Nonbullous impetigo physical findings are as follows
 The first noticeable abnormality is a red macule or papule, from 2-5 mm in size.
 The characteristic lesion is a fragile vesicle or pustule that readily ruptures and becomes a honey-yellow, adherent,
crusted papule or plaque smaller than 2 cm and with minimal or no surrounding redness.
 Lesions develop on either normal or traumatized skin or are superimposed on a preexisting skin condition (eg, scabies,
varicella, atopic dermatitis) and can spread rapidly.
 Lesions are located around the nose, mouth, and exposed parts of the body (eg, arms, legs), sparing the palms and
soles.
 Localized lymphadenopathy is usually present, and nodes may be tender.
 If left untreated, lesions spread by autoinoculation then spontaneously resolve after a few weeks without scarring.
 Rarely, pedal edema and hypertension may be noted in an individual with nonbullous impetigo. Both are signs of renal
dysfunction most likely resulting from glomerulonephritis.

D. MOLLUSCUM CONTAGIOSUM

History
 Most patients are asymptomatic; some complain of pruritus, tenderness, and pain.
 Some develop eczema around lesions (10% in series of 95 and 200 cases).
 The incubation period ranges from weeks to months (14-50 d). Case studies of vertically transmitted molluscum contagiosum
describe a relatively short incubation period, with infections appearing on infants between ages of a few days to 6 weeks following maternal
exposure
 If patients have eczema or other diseases altering skin barrier function, molluscum may spread more rapidly in affected areas.
Physical
Physical findings generally are limited to the skin, but cases have reported findings on the eyelids and conjunctiva.

 Skin - Primary lesion of molluscum contagiosum


o Firm, smooth, umbilicated papules, usually 2-6 mm in diameter (range 1-15 mm), may be present in groups or may be
widely disseminated on the skin and mucosal surfaces.
o The lesions can be flesh-colored, white, translucent, or even yellow in color.
o The number of lesions varies from 1-20 up to hundreds in some reports.
o Some lesions become confluent to form a plaque.
o Lesions generally are self-limited but can persist for several years.
 Skin - Distribution of molluscum contagiosum
o In children, papules are located mainly on the trunk and extremities.
o In adults, lesions often are located on the lower abdominal wall, inner thighs, pubic area, and genitalia.
o Although rarely found in the mouth or on the palms and soles, cases of molluscum contagiosum involving the oral
mucosa, including the lips, buccal mucosa, hard palate, retromolar pad, and tongue, have been reported.

E. PAPULAR UTRICARIA
History
Patients report usually chronic or recurrent episodes of a papular eruption that tends to occur in groups or clusters associated with intense
pruritus.

Children, adult males, nonlocals, and those belonging to urban or periurban areas may be more vulnerable to papular urticaria. [16] The most
common first appearance is of papules and urticarial plaques in clusters over exposed and covered parts of the body.

Physical
Papular urticaria is characterized by crops of symmetrically distributed pruritic papules and papulovesicles. The lesions can also appear in an
area localized to the site of insect bites. Papules may occur on any body part, but they tend to be grouped on exposed areas, particularly the
extensor surfaces of the extremities. Scratching may produce erosions and ulcerations. Secondary impetigo or pyoderma is common.
22
Causes
A hypersensitivity reaction to the bites of mosquitoes, fleas, bedbugs, and other insects causes papular urticaria. It is unusual to identify an
actual culprit in any given patient.
F. PSORIASIS
G.
History
Symptoms of psoriasis may include the following:
Worsening of a long-term erythematous scaly area,Sudden onset of many small areas of scaly redness,Recent streptococcal throat
infection, viral infection, immunization, use of antimalarial drug, or trauma,Family history of similar skin condition,Pain (especially in
erythrodermic psoriasis and in some cases of traumatized plaques or in the joints affected by psoriatic arthritis),Pruritus (especially in eruptive,
guttate psoriasis),Afebrile (except in pustular or erythrodermic psoriasis in which the patient may have high fever),Dystrophic nails,Long-
term rash with recent presentation of joint pain,Joint pain without any visible skin findings
The skin almost always is affected before the eyes. Ocular findings occur in approximately 10% of patients. The most common ocular
symptoms are redness and tearing due to conjunctivitis or blepharitis.
The nonocular symptoms are related to rash and psoriatic arthritis. The rash can be uncomfortable or even painful. Psoriatic arthritis can cause
stiffness, pain, throbbing, swelling, or tenderness of the joints. The distal joints, such as the fingers, toes, wrists, knees, and ankles, are most
often affected.
Physical Examination
Findings on physical examination depend on the type of psoriasis present.
The most common skin manifestations are scaling erythematous macules, papules, and plaques. Typically, the macules are seen first, and
these progress to maculopapules and ultimately well-demarcated, noncoherent, silvery plaques overlying a glossy homogeneous erythema.
The area of skin involvement varies with the form of psoriasis.
Chronic stationary psoriasis (psoriasis vulgaris) is the most common type of psoriasis. This involves the scalp, extensor surfaces, genitals,
umbilicus, and lumbosacral and retroauricular regions.
Plaque psoriasis is characterized by raised, inflamed lesions covered with a silvery white scale. The scale may be scraped away to reveal
inflamed skin beneath. This is most common on the extensor surfaces of the knees, elbows, scalp, and trunk.
Guttate psoriasis presents as small salmon-pink papules, 1-10 mm in diameter, predominately on the trunk; the lesions may be scaly (see the
image below). It frequently appears suddenly, 2-3 weeks after an upper respiratory infection (URI) with group A beta-hemolytic streptococci.

Inverse psoriasis occurs on the flexural surfaces, armpit, groin, under the breast, and in the skin folds. It is characterized by smooth, inflamed
lesions without scaling due to the moist nature of the area where this type of psoriasis is located.
Pustular psoriasis presents as sterile pustules appearing on the palms and soles or diffusely over the body. Pustular psoriasis may cycle through
erythema, pustules, then scaling. The diffuse variant is termed von Zumbusch variant, which is accompanied by fever and intense ill feeling in
addition to the widespread pustules. Acrodermatitis continua of Hallopeau is considered a form of pustular psoriasis that affects the hands and
feet. It may prove resistant to topical and other therapies.
Erythrodermic psoriasis presents as generalized erythema, pain, itching, and fine scaling; various pustular forms also exist. It typically
encompasses nearly the entire body surface area. It may be accompanied by fever, chills, hypothermia, and dehydration secondary to the large
body surface area involvement. Patients with severe pustular or erythrodermic psoriasis may require hospital admission for metabolic and pain
management. Older patients with erythrodermic psoriasis may experience cardiac instability and hypotension due to massive vascular shunting
in the skin.
Scalp psoriasis affects approximately 50% of patients. It presents as erythematous raised plaques with silvery white scales on the scalp.
Nail psoriasis may cause pits on the nails, which often become thickened and yellowish in color. Nails may separate from the nail bed. Psoriatic
nails may be indistinguishable from fungal nails and, at the same time, may be more prone to developing onychomycosis because of the nail
separation and subungual debris.
23 The arthritis is usually in the hands and feet and, occasionally, the
Psoriatic arthritis affects approximately 10-30% of those with skin symptoms.
large joints. It produces stiffness, pain, and progressive joint damage.
Oral psoriasis may present with whitish lesions on the oral mucosa, which may appear to change in severity daily. It may also present as severe
cheilosis with extension onto the surrounding skin, crossing the vermillion border. Geographic tongue is considered by many to be an oral form of
psoriasis.
H. SCABIES

History
The historical aspects of scabies infestations are quite reliable in suggesting the diagnosis. Lesion distribution, intractable pruritus that is worse at
night, and similar symptoms in close contacts should immediately rank scabies at the top of the clinical differential diagnosis.

Lesion distribution differs in adults and children. Adults manifest lesions primarily on the flexor aspects of the wrists, the interdigital web spaces of
the hands, the dorsal feet, axillae, elbows, waist, buttocks, and genitalia. Pruritic papules and vesicles on the scrotum and penis in men and areolae
in women are highly characteristic.

Infants and small children may develop lesions diffusely, but unlike adults, lesions are common on the face, scalp, neck, palms, and soles. All
cutaneous sites are susceptible in immunocompromised and elderly patients, who often have a history of a widespread, pruritic eczematous
eruption.

Unlike adults, who rarely present with facial and neck involvement, this presentation is fairly typical in children.
In very young children and infants, a widespread eczematous eruption primarily on the trunk is common, as in the image below.
Physical
Clinical findings include both primary and secondary lesions. Primary lesions are the first manifestation of the infestation, and these typically include
small papules, vesicles, and burrows. Secondary lesions are the result of rubbing and scratching, and they may be the only clinical manifestation of
the disease. If so, the diagnosis must be inferred by the history, lesion distribution, and accompanying symptoms.

Primary scabies lesions


The distribution is highly characteristic in typical cases.

Burrows are a pathognomonic sign and represent the intraepidermal tunnel created by the moving female mite. They appear as serpiginous,
grayish, threadlike elevations ranging from 2-10 millimeters long

History
Intermittent, active phases of seborrheic dermatitis manifest with burning, scaling, and itching, alternating with inactive periods. Activity is
increased in winter and early spring, with remissions commonly occurring in summer.
Active phases of seborrheic dermatitis may be complicated by secondary infection in the intertriginous areas and on the eyelids.
H.SEBBORHEIC
Candidal overgrowth is commonDERMATITIS
in infantile napkin dermatitis. Such children may have a diaper dermatitis variant of seborrheic dermatitis or
psoriasis.
Generalized seborrheic erythroderma is rare. It occurs more often in association with AIDS, [2, 3] congestive heart failure, Parkinson disease, and
immunosuppression in premature infants.
Physical
The scalp appearance of seborrheic dermatitis varies from mild, patchy scaling to widespread, thick, adherent crusts. Plaques are rare. From the
scalp, seborrheic dermatitis can spread onto the forehead, the posterior part of the neck, and the postauricular skin, as in psoriasis. Note the
images below.
Seborrheic dermatitis skin lesions manifest as branny or greasy scaling over red, inflamed skin. Hypopigmentation is seen in blacks. Infectious
eczematoid dermatitis, with oozing and crusting, suggests secondary infection. A seborrheic blepharitis may occur independently.
Distribution follows the oily and hair-bearing areas of the head and the neck, such as the scalp, the forehead, the eyebrows, the lash line, the
24
nasolabial folds, the beard, and the postauricular skin. An extension to submental skin can occur. Presternal or interscapular involvement is more
common than nonscaling intertrigo of the umbilicus, axillae, inframammary and inguinal folds, perineum, or anogenital crease, which also may be
present.
Two distinct truncal patterns of seborrheic dermatitis can occasionally occur. An annular or geographic petaloid scaling is the most common. A
rare pityriasiform variety can be seen on the trunk and the neck, with peripheral scaling around ovoid patches, mimicking pityriasis rosea.
I. TINEA CORPORIS

History
Symptoms, contact history, recent travel, and international residence are relevant clues in the history of a person with tinea corporis. Infected
patients may have variable symptoms. Patients can be asymptomatic.
o
A pruritic, annular plaque is characteristic of a symptomatic infection. Patients occasionally can experience a burning
sensation.
o
HIV-positive or immunocompromised patients may develop severe pruritus or pain.
 Tinea corporis may result from contact with infected humans, animals, or inanimate objects. The history may include occupational (eg,
farm worker, zookeeper, laboratory worker, veterinarian), environmental (eg, gardening, contact with animals), or recreational (eg, contact
sports, contact with sports facilities) exposure.
 A few clinical variants are described, with distinct presentations.

o
Majocchi granuloma, typically caused by T rubrum, is a fungal infection in hair, hair follicles, and, often, the surrounding
dermis, with an associated granulomatous reaction. Majocchi granuloma often occurs in females who shave their legs.
o
Tinea corporis gladiatorum is a dermatophyte infection spread by skin-to-skin contact between wrestlers. [4, 5]
o
Tinea imbricata is a form of tinea corporis found mainly in Southeast Asia, the South Pacific, Central America, and South
America. It is caused by Trichophyton concentricum.[6]
o
Tinea incognito is tinea corporis with an altered, nonclassic presentation due to corticosteroid treatment. [7]
Physical
 History
Tinea corporis can manifest in a variety of ways.
o The following are the most
Typically, common
the presenting
lesion begins as ansymptoms of varicella:
erythematous, scaly plaque that may rapidly worsen and enlarge, as shown in the image
below. e.
o  Low-grade fever preceding skin manifestations by 1-2 days
Following central resolution, the lesion may become annular in shape, as is shown in the image below.
o  Complaints of abdominal
As a result pain by some
of the inflammation, children
scale, crust, papules, vesicles, and even bullae can develop, especially in the advancing
 border Pleomorphic rash, usually starting on the head and trunk and spreading to the rest of the body
o
 Rarely,
Typically, tinea corporis
complaints pruritus as purpuric macules, called tinea corporis purpurica. [9] One report describes 2 cases of
can present
of intense

o
 tinea corporis purpurica
Headache resulting from self-inoculation with Trichophyton violaceum
Infections due to zoophilic or geophilic dermatophytes may produce a more intense inflammatory response than those
 Malaise
caused by anthropophilic microbes.
o  Anorexia
HIV-infected or immunocompromised patients often have atypical presentations including deep abscesses or a disseminated
 skin infection.
Cough and coryza
 Sore throat
Children with eczema or dermatitis may have severe skin manifestations during varicella.

J. should
The history VARICELLA
elicit if aZOSTER
recent outbreak of chickenpox in the community has occurred and if any exposure to varicella at school,
daycare, or among family members has occurred. Most patients have a history of exposure in one or more of these three settings. It should
also be noted whether the child has previously received varicella vaccine or if the child is immunocompromised (including recent systemic
steroid use) to help guide management.
Examination of rash
The diagnosis of varicella is made upon observation of the characteristic chickenpox rash. This rash appears in crops. Skin lesions initially
appear on the face and trunk, beginning as red macules and progressing over 12-14 days to become papular, vesicular, pustular, and
finally crusted. New lesions continue to erupt for 3-5 days. Lesions usually crust by 6 days (range 2-12 d), and completely heal by 16 days
(range 7-34 d). Prolonged eruption of new lesions or delayed crusting and healing can occur with impaired cellular immunity.

An otherwise healthy child usually has 250-500 lesions but may have as few as 10 or as many as 1500. The lesions predominate in central
skin areas and proximal upper extremities with relative sparing of distal and lower extremities but spread to other skin areas. Some lesions
may appear in the oropharynx. Eye lesions are rare. 25
Each lesion starts as a red macule and passes through stages of papule, vesicle, pustule, and crust. The vesicle on a lesion’s
erythematous base leads to its description as a pearl or dewdrop on a rose petal. Vesicles may occur on mucous membranes and break
down to form shallow aphthous ulcers. Vesicles can be hemorrhagic. Redness or swelling around a lesion should lead to suspicion of
bacterial superinfection. Dermatomal distribution of lesions is characteristic of reactivation rather than primary infection.
26. SEBORRHEIC DERMATITIS
27. SCABIES
28. ATOPIC ECZEMA
29. VARICELLA Z
30. PSORIAIS

EMQ 7

THEME: TREATMENT OF SEXUALLY TRANSMITTED DISEASES

(MOST ANSWERS CAME FROM NUNES LECTURE ON STDS FOR ANY1 WHO WAS CURIOUS)

31) A 19 yr old commercial sex worker presents with a hard painless sore on the genitalia

(Dx = Syphylis!!! Because see table below)

ANS = J

26
Treatment of primary, secondary, and early latent <1 year syphilis:
- Benzathine penicillin G 2.4 million units IM for 1 dose
- Penicillin-allergic patients: Doxycycline 100 mg PO b.i.d. for 2 weeks or tetracycline 500 mg PO q.i.d.
for 2 weeks; Ceftriaxone 1 g IM or IV daily for 8-10 days is used by some

32) A37 yr old male taxi driver complains of urethral discharge. You decide to treat him as uncomplicated
urethritis. You give him norfloxacin and one other drug.

(BTW uncomplicated urethritis aka non-gonococcal urethritis)

(Dx= well they already gave you the diagnosis as uncomplicated urethritis so....)

Ans =H

27
33) A 21 yr old university student presents with recurrent episodes of herpes genitalis.

(Dx = again the diagnosis is there)

Ans = D

Genitalis Herpes

Etiology: Herpes Simplex type 2.

 HSV-2 spreads via the skin to sensory nerve endings- retrograde to sensory ganglia-replication- and
antegrade to skin.
 HSV-2 recurs more frequently than HSV-1.

Clinical features:

 Primary herpes genitalis- systemic illness, may present as aseptic meningitis /urinary retention.
 Painful vesicles on penis, labia, vulva, cervix, anus
 Recurrent HSV-2 will occur in 80% of primary infections (penile/ vaginal vesicular lesions).
 “Attack” usually lasts 6-10 days.

Treatment:

Acyclovir is the drug of choice for most herpes infections.


Acyclovir is available in many formulations as:-
(a) I.V. for HSV infection in normal and immunocompromised patients
(b) Oral for treatment and long term suppression of muco cutaneous herpes and prophylaxis of HSV in
immunocompromised patients
(c) Cream for HSV infection of the skin and mucous membranes
(d) Ophthalmic ointment

Other older agents: such as idoxuridine, trifluorothymidine, Vidarabine (ara-A) are highly toxic and is
suitable for topical use for ophthalmic infection only.

34) a 35 yr old waitress presents with lower abdominal pain, fever, cervical excitation, and vaginal
discharge. You treat with doxycycline, metranidazole and one other .

(Dx = PID)

ANS =A

PID - Definition: infection involving Fallopian tubes, one or both ovaries, uterus and pelvic peritoneum

Criteria for diagnosis of PID:

28
 Abdominal tenderness
 Cervical excitation tenderness

 Adnexal tenderness

Plus one of below:


Temperature > 38 oC

Leucocytosis > 10,000/mm3

Pelvic abscess or inflammatory complex

Gram’s stain of endocervix for Neisseria gonorrhea (GN diplococci)

Purulent endocervical discharge with leukocytes on smear

Mononuclear smear positive for Chlamydia trachomatis

Purulent material on culdocentesis

Treatment
2 Regimens (NICE)
1) Ofloxacin 400mg po bd & Metranidazole 400 mg bd x2/52
2) IM Ceftriaxone 250mg and Doxycycline 100mg po and Metranidazole 400mg po bd x 2/52

35) A 29 yr old secretary who has not had sex in the last 3 months complains of a fishy-smelling
discharge. Wet prep shows clue cells.

(Dx= Bacterial Vaginosis b/c the clue cells gave you a clue!!)

Ans= E

29
EMQ 8 :36-40 36) Diagnosis Alziemer’s disease treated with DONEZEPIL. Patients present with
decreased memory and cognition, behavioural change, hallucinations, delusions, apathy, depression and
irritability. Treat with acetylcholinesterase inhibitors eg. DONEZEPIL, rivastigmine, galantamine. OHCM
pg. 480

37) Diagnosis Manic episode treated with CARBAMAZEPINE. Signs of mania include:

1) mood- irritability, euphoria, lability

2) cognition- grandiosity, flight of ideas, racing thoughts, distractibility, poor concentration

3) behaviour- rapid speech, hyperactivity, hypersexuality, decreased sleep, extravagance

4) psychotic- hallucinations, delusions

Treated with mood stabilizers eg, benzodiazepines, valproate and lithium.OHCS pg. 354

38) Diagnosis Anxiety disorder treatment PAROXETINE. Symptoms of anxiety include: tension agitation,
feelings of impending doom, trembling, poor concentration, hyperventilation, chest pain, headaches,
pappitations, sweating, globus hystericus, difficulty sleeping, repetitive thoughts. Treatment
psychotherapy with anxiolytics eg. Diazepam, PAROXETINE.OHCS pg. 344

39) Diagnosis Alcohol withdrawal treated with CHLORDIAZEPOXIDE. Starts 10-72hrs. after last drink.
Signs include: tachycardia, hypotension, tremor, confusion, fits, hallucinations(visual or tactile). Treated
with CHLORDIAZEPOXIDE and diazepam. DON’T WORRY I KNOW THERE ARE A FEW PEOPLE IN GROUP A
WHO WILL NEED SOME OF THIS AFTER EXAMS YOU CAN CHECK MEH I ALREADY HAVE IN STOCK!!!!!!
OHCM pg. 274

30
40) Diagnosis Smoking cessation treatment BUPROPION. OHCS pg. 512

MCQ s
1. A
OHCM Pg 252: Cirrhosis- commonly caused by chronic alcohol abuse, HBV, HCV infection. Complications
include hepatic failure, portal hypertension and increased risk of hepatocellular carcinoma. Ascitic tap
should be performed and fluid sent for urgent MCS (neutrophils >250/mm 3 indicates spontaneous
bacterial peritonitis (common organisms are E coli, Klebsiella and streptococcus.

2. A
Lab results suggest a pancytopenia! Options B, D and E would give low Hb levels. Option C would give a
pancytopenia but this would not occur secondary to treatment with radioiodine.

Pernicious is an autoimmune condition associated with other autoimmune disorders such as Graves.
Tests reveal low Hb levels and in severe cases, low WCC and platelets as well. Also, low serum B12,
reticulocytes decreased or normal, hypersegmented polymorphs, megaloblasts in marrow, parietal cell
antibodies, intrinsic factor antibodies. (Check 2010 EMQ 3 for more info on different types of anaemias).

3. C
Positive gram stain on joint aspirate- most specific. (Joint aspiration sent for WCC, polarized light
microscopy and gram stain and culture).

Monosodium urate crystals are found in gout; polarized light microscopy of synovial fluid shows
negatively birefringent crystals.

Elevated peripheral WBC count is not specific to septic arthritis, neither is elevated ESR.

4. C (not sure… cud be B)


Obesity suggests increasing insulin resistance. Family history- mother.

MODY is a rare autosomal dominant form of type 2 DM affecting young people with a positive family
history.

But type 2 DM may also occur in teenagers, associated with obesity and there is a strong genetic
influence.

5. C (most likely feature… not cause)

Left 3rd cranial nerve paralysis:

- dilated left pupil that is unreactive to light


- normal consensual light reflex on the right

- unreactive to accommodation

31
- left ptosis

- divergent strabismus (eye down and out)

(Absence of sweating may occur in a Horner’s syndrome)

6. C

B blockers (enough to decrease the pulse to <60, continue for at least one year) reduces mortality
from all causes by 25% in patients who have had a previous MI.

7. D

Acute glaucoma- primary angle closure glaucoma, more common in females >40yr (toronto
notes have >70yr). Patient likely to be long sighted with small anterior chamber. C/F: abrupt
increase in pressure leads to red painful eye, photophobia, watering of eye reduced visual acuity,
cornea cloudy, pupil oval fixed and dilated. Patient may have nausea, vomiting and abdominal
pain. Treatment includes acetazolamide and B blockers (reduce aqueous secretion) and
pilocarpine (constricts pupil and draws iris out of angle). Iridotomy/iridectomy can also be done
with a YAG laser or surgically.

Episcleritis- inflammation of superficial layer of sclera causing mild discomfort and is self
limiting.

Acute bacterial conjuctivitis- redness of eye, discharge, ocular irritation. Coomon causative
organisms: staph, strep, pneumococcus, moraxella and haemophilus. Consider chlamydia in
neonates and neisseria in neonates and sexually active ppl. Self limiting.

Subconjuctival haemorrhage- aymptomatic besides blood beneath conjuctiva.

SAH- thunderclap headache

8. most likely cause of localized non scarring alopecia –

Ans C alopecia areata – sharply defined non inflamed bald patches usually on scalp.
Pathognomonic ‘exclamation mark’ hairs seen (broken off haris 3-4mm long). Also affects
eyebrows, eyelashes and beard. Usually regrows spontaneously.

Lichen planus - chronic mucocutaneous disease that affects the skin, tongue, and oral mucosa.
The disease presents itself in the form of papules,[1] lesions, or rashes. The typical rash of lichen
planus is well-described by the "5 Ps": well-defined pruritic, planar, purple, polygonal papules.
The commonly affected sites are near the wrist and the ankle. The rash tends to heal with
prominent blue-black or brownish discoloration that persists for a long time. Besides the typical
lesions, many morphological varieties of the rash may occur. The presence of cutaneous lesions

32
is not constant and may wax and wane over time. Oral lesions tend to last far longer than
cutaneous lichen planus lesions.

Tinea capitis – patchy hair loss with some scaling. Due to fungal infection, usually in children.
Treatment with oral terbinafine, itraconazole, griseofulvin. Topical therapy , eg antifungal
shampoo, and arachis oil is used to remove crusting.

Carbuncle - collection of multiple infected hair follicles. It is an abscess, just like afuruncle is an
abscess but a carbuncle is a much more serious infection. Whereas a furuncle is an infection of a
hair follicle and the surrounding tissue, a carbuncle is actually several furuncles that are densely
packed together.

9. 33 yo male, painless haematuria, hearing impairment, siblings similar disease

Ans – D alport’s syndrome - (aka Hereditary Nephritis) is a genetic disorder[1] characterized


by glomerulonephritis, endstage kidney disease, and hearing loss.[2] Alport syndrome can also
affect the eyes (lenticonus). The presence of blood in the urine (hematuria) is almost always
found in this condition. Mutations certain genes in collagen synthesis prevent the proper
production or assembly of the type IV collagen network, which is an important structural
component of basement membranes in the kidney, inner ear, and eye. Basement membranes
are thin, sheet-like structures that separate and support cells in many tissues. When mutations
prevent the formation of type IV collagen fibers, the basement membranes of the kidneys are
not able to filter waste products from the blood and create urine normally, allowing blood and
protein into the urine. The abnormalities of type IV collagen in kidney basement membranes
cause gradual scarring of the kidneys, eventually leading to kidney failure in many people with
the disease. Progression of the disease leads to basement membrane thickening and gives a
"basket-weave" appearance from splitting of the lamina densa.

As there is no known cure for the condition, treatments are symptomatic. Patients are advised
on how to manage the complications of kidney failure, and the proteinuria that develops is
often treated with ACE inhibitors, although they are not always used simply for the elevated
blood pressure.[12]

Once kidney failure has developed, patients are given dialysis or can also benefit from a kidney
transplant, although this can also cause problems. The body may reject the new kidney as it
contains normal type IV collagen, which may be recognized as foreign by the immune system
10 thyroid question. Pt clinically hyperthyroid – anxiety, palpitations, tachycardic. T3 and T4
elevated. TSH undetectable.

Ans C Graves’ disease

33
Factitious thyrotoxicosis – pt induces this by consuming excessive amounts of thyroxine. There
is suppression of tsh secretion and the t4:t3 ratio is increased to above 70:1.

T3 thyrotoxicosis – normal T4, raised T3, undetectable tsh

Subacute thyroiditis - form of thyroiditis that can be a cause of


both thyrotoxicosis and hypothyroidism. 2 types ; de quervain’s thyroiditis , and subacute
lymphocytic thyroiditis.

Question on de quervain’s later on.

Subacute lymphocytic thyroiditis - also known as silent thyroiditis or painless thyroiditis. It


features a small goiter without tenderness. This condition tends to have a phase
of hyperthyroidism followed by a return to a euthyroid state, and then a phase
of hypothyroidism, followed again by a return to the euthyroid state. The time span of each
phase can vary; however, each phase usually lasts 2-3 months. Subacute lymphocytic thyroiditis
can only be diagnosed correctly by taking a radioactive iodine uptake test, or RAIU test.[1][3]

During the hyperthyroid phase, iodine uptake is suppressed, while during the hypothyroid
phase, uptake is increased.
Treatment is symptomatic.

11. pituitary macroadenoma – lesion at optic chiasm

Ans D bitemporal hemianopia

12. 70 yo with stroke, feature most indicative of poor prognosis

Ans A level of consciousness

13. polycystic ovarian syndrome

Ans B forerunner to type 2 dm

Sequelae of pcos: dyslipidaemia and cardiovascular disease, hypertension, GDM and DM type
II, endometrial ca

14. pain in neck following a flu like illness. Hyperthyroid symptoms

Ans C de quervain’s thyroiditis

Some cases may be viral in origin, perhaps preceded by an upper respiratory tract infection.
Viral causes include Coxsackie virus, mumps and adenoviruses. Some cases develop
postpartum.

34
Patients will experience a hyperthyroid period as the cellular lining of colloid spaces fails,
allowing abundant colloid into the circulation, with neck pain and fever. Patients typically then
become hypothyroid as the pituitary reduces TSH production and the inappropriately released
colloid is depleted before resolving to euthyroid. The symptoms are those
ofhyperthyroidism and hypothyroidism. In addition, patients may suffer from painful dysphagia.
There are multi-nucleated giant cells on histology. Thyroid antibodies can be present in some
cases. The clinical presentation during the hyperthyroid phase can mimc that of diffuse toxic
goiter (Grave's disease). In such cases, a radionuclide thyroid uptake and scan can be helpful,
since subacute thyroiditis will result in decreased isotope uptake, while Grave's disease will
generally result in increased uptake. Distinguishing between these two diseases is important,
since Grave's disease can be treated with radioiodine therapy, but subacute thyroiditis is usually
self-limited and is not treated with radioiodine.

Treatment is with beta blockers and NSAIDS.

15. A 67-year-old female has longstanding rheumatoid arthritis. She is on steroids as well
as intermittent NSAIDS. Two weeks ago, she notices that her ankles were swelling. She
consults her general practitioner who diagnosed the nephrotic syndrome and ultrasound
showed enlarged kidneys. What is the most likely cause of this condition?

(A) RENAL AMYLOIDOSIS

(b) chronic interstitial nephritis

(c) analgesic nephropathy

(d) chronic pyelonephritis

(e) focal and segmental glomerulosclerosis

A variety of renal disorders can occur in patients with rheumatoid arthritis (RA), due to the
underlying disease, to drugs used to treat the inflammatory process, and to concurrent
renal disease unrelated to RA, which is more likely in elderly patients with significant
comorbid conditions.
The most common disorders associated with RA are membranous nephropathy, secondary
amyloidosis, a focal, mesangial proliferative glomerulonephritis, rheumatoid vasculitis, and
analgesic nephropathy [In this case NSAID nephropathy is unlikely due to its intermittent
use]. Other disorders, such as IgA nephropathy and minimal change disease have also been
reported in patients with RA, although this may represent coincidental disease.

16. A 46-year-old male who has consumed a pint of rum daily for 18 years, presents with
right upper quadrant pain. The pain is of four days duration. He gives a history of taking
two extra strength acetaminophen tablets every four hours for the past week to relieve
headache. There is no history of jaundice or gallstones. The patient had a blood transfusion
20 years ago when he was hospitalized for a motor vehicle accident. There is no history of

35
recent exposure to hepatitis. Physical examination revealed an enlarged tender liver with a
span of 18cm. the most likely diagnosis is:

(A) ALCOHOLIC HEPATITIS


(b) acute cholecystitis
(c) acetaminophen hepatotoxicity
(d) acute viral hepatitis B
(e) acute viral hepatitis C
Risk Factors for Alcoholic Liver Disease :

Male, Chronic alcohol consumption - 18 year history of a pint daily

The Three Stages of Alcoholic Liver Disease Are:


 Fatty Liver
 Alcoholic Hepatitis (Hepatomegaly)
 Cirrhosis (Shrunken liver)
2 extra strength acetaminophen tablets are a normal dose – can be taken q4-6 hours. Not
consistent with overdose.

No history of jaundice or gallstones – r/o Acute cholecystitis

R/o hepatits B and C – no recent exposure, and would not get acute hepatitis 20 years after
transfusion. Acute viral hepatitis is more likely to be asymptomatic in younger people.
Symptomatic individuals may present after convalescent stage of 7 to 10 days, with the
total illness lasting 2 to 6 weeks.

17. A 75-year-old chronic smoker is experiencing rapid weight loss, hyperpigmentation


and muscle weakness associated with marked hypokalaemia. The most likely cause of the
clinical presentation is:

(a) metastatic disease to the brain

(b) metastatic disease to the kidney causing potassium wasting

(c) superimposed tuberculosis

(D) ACTH PRODUCTION BY THE TUMOUR

(e) myasthenia gravis

Chronic Smoker with rapid weight loss -- > Lung Cancer

Neuroendocrine tumours of the lung may be associated with the ectopic


adrenocorticotrophin (ACTH) syndrome and may synthesize and secrete ACTH-related
peptides in the absence of the syndrome.

36
Pro-opiomelanocortin (POMC) gene in neuroendocrine lung tumours

Melanin -> Responsible for hyperpigmentation

ACTH -> Cortisol -> Responsible for Cushing’s features – Muscle weakness and
hypokalaemia

18. A 25-year-old man, who has been previously well, presents with acute onset of dyspnea.
On examination of the respiratory system he has tracheal deviation to the right. There is
reduced expansion, hyper-resonant percussion, reduced breath sounds and reduced tactile
vocal fremitus on the left side of the chest. His oxygen saturation is 88% breathing air. The
blood pressure is 80mmHg systolic. Of the following, the most appropriate emergency
management would be to:

(a) arrange a spiral CT scan of the chest

(B) ASPIRATE THE LEFT SIDE OF THE CHEST

(c) give antibiotics intravenously

(d) give increased inspired oxygen with 24% facemask

(e) treat with non-invasive positive pressure ventilation by face mask

Needle Thoracostomy
Classical management of tension pneumothorax is emergent chest decompression with
needle thoracostomy. A 14-16G intravenous cannula is inserted into the second rib space in
the mid-clavicular line. The needle is advanced until air can be aspirated into a syringe
connected to the needle. The needle is withdrawn and the cannula is left open to air. An
immediate rush of air out of the chest indicates the presence of a tension pneumothorax.
The manoeuver essentially converts a tension pneumothorax into a simple pneumothorax.

19. An 80-year-old male has atrial fibrillation and hypertension. Which therapy would best
reduce his chances of having an ischaemic stroke?

(A)WARFARIN
(b) digoxin
(c) aspirin
(d) furosemide
(e) clopidogrel
Anticoagulation can be achieved through a number of means including the use of aspirin,
heparin, warfarin and dabigatran. Which method is used depends on a number issues
including: cost, risk of stroke, risk of falls, compliance, and speed of desired onset of
anticoagulation.

37
Anticoagulant therapy with warfarin is now established as effective thromboprophylaxis
against stroke in atrial fibrillation, in high-risk persons. Aspirin is indicated in moderate-
risk persons or if warfarin is contraindicated.

Most strokes occur in patients assigned to warfarin therapy with INR values under 2·0. At
the other end of spectrum, the bleeding risk was higher in patients with INR values over
3·0. The target INR for anticoagulation in non-valvular AF should therefore be between 2·0
and 3·0, providing maximum thromboprophylaxis with minimum bleeding risk.

20. In the assessment of patients with hypertension, the presence of coarctation of the
aorta is best identified clinically by:

(a) sustained hypertension

(b) unequal arterial arm pules

(C) RADIOFEMORAL IMPULSE DELAY

(d) absence of arterial foot pulses

(e) slow rising carotid artery pulsation

Coarctation of the aorta is an important and treatable cause of secondary hypertension.

Palpation of the brachial and femoral pulses simultaneously will show decreased and
delayed or absent femoral pulses. On measurement of blood pressure from arms and legs, a
pressure difference of more than 20 mm Hg in favor of the arms may be considered as
evidence for coarctation of the aorta.

21. Of the following, a flapping tremor with a foetor oris best occurs in:

(a) alcohol withdrawal

(B) LIVER FAILURE

(c) pancreatitis

(d) chronic obstructive airway disease

(e) diabetic ketoacidosis

Causes of a flapping tremor (asterixis):

 Liver failure
 Renal failure
 Ventilatory failure
 Carbon dioxide retention
 Acute focal parietal or thalamic lesions

Foetor oris:

38
Foetor hepaticus -> stale ‘mousy’ smell of the volatile amine, methyl mercaptan

Diabetic Ketoacidosis -> Sweet acetone breath

Oral causes – poor hygiene, food stuck between teeth, acute rhinosinusitis, tonsillitis,
impacted wisdom teeth, etc.

The only option consistent with both flapping tremor and foetor oris is liver failure

22. Answer: C
Massive splenomegaly: CML, myelofibrosis, malaria, leishmaniasis, gaucher’s syndrome
Moderate: Infection(EBV, IE, TB, and those above), portal hypertension, haematologic
(haemolytic anaemia, leukaemia, lymphoma, RA, SLE, sarcoidosis

23. Answer: E
Ref 2009 P1 emq

24. Answer: E
Pseudomembranous colitis is a cause of antibiotic-associated diarrhea (AAD) an infection of
the colon. It is often caused by the bacterium Clostridium difficile (gram +ve anaerobic
sporeforming rod) * clindamycin and cephalosporins account for a large percentage of causative
agents. Rx: 1st metronidazole , alternative vanco.

25. Answer: B
Inferior MI: II, III, aVF
Anteroseptal: V1- V4
Anterolater: V4- V6, I, aVL
Posterior: tall R and ST ↓ V1-V2

26. Answer: C
Horner’s Syndrome: Pancoast tumor: smoker, left ptosis, miosis, enophthalmos, anhidrosis

27. Answer: C
Liver enzymes elevated in keeping with both abcess and viral hep however symptoms (fever,
body pains, dec appetite, right pleuritic chest pain) are in keeping with liver abscess

28. Answer: C

39
29. E
Lung: Bronchiectasis, Emphysema, fibrosing alveolitis, Lung abscess, Bronchial ca(not small cell),
Mesothelioma

Cardiac: Endocarditis, Cyanotic Congenital Heart disease, Atrial Myxoma

GIT: IBD, GI lymphoma, Cirrhosis, Malabsorption

Rare: familial, thyroid acropachy

30) A
Acute necrosis of renal tubules due to ischemic or toxic insult
_ Ischemic: Shock, trauma, sepsis, hypoxia
_ Toxic: IV contrast media, aminoglycosides, rhabdomyolysis, and tumor lysis
COURSE
_ Patients present with dramatic renal failure
_ Most survive and recover normal renal function. Failure lasts 1 to 2 weeks, during which intensive care
is required.
DIAGNOSIS
_ Muddy brown granular (rbc casts)
_ High urine sodium (>40 mmol/L)
_Urine osmolarity <350 mosm/L
_ FE Na is > 1%.
31) C

Anaemia of Chronic Renal Failure

Relative deficiency of epo. NB. Epo levels are in normal range in plasma but they are inappropriately low
for the degree of anaemia. Uraemia suppresses bone marrow

Recomb human epo is best Mx option. Target Hb is between 10-12. Complications are hypertension and
increased blood coagulability increased thrombosis of fistulae. Epo is ineffective if Fe deficient, active
inflammation or malignancy or aluminium overload (as occurs in dialysis). So admin Fe supplements to
keep ferritin>100ug/l.

40
32) C

Quinolone or cephalsporin or gentamicin x 7-14 days

105cfu - MSU

103cfu-Catheter

Any organisims -Surapubic aspirate

If pt is symptomatic with neutrophils in urine less organisms is significant.

33) A

NSAIDS

 Gastro-intestinal discomfort, nausea, diarrhoea, and occasionally bleeding and ulceration.

Systemic as well as local effects of NSAIDs contribute to gastro-intestinal damage; taking oral
formulations with milk or food, or using enteric-coated formulations, or changing the route of
administration may only partially reduce symptoms such as dyspepsia. Hypersensitivity reactions
(particularly rashes, angioedema, and bronchospasm).

 Headache, dizziness, nervousness, depression, drowsiness, insomnia, vertigo, hearing


disturbances such as tinnitus, photosensitivity, and haematuria. Blood
 Fluid retention may occur (rarely precipitating congestive heart failure); blood pressure may be
raised.
 Renal failure may be provoked by NSAIDs, especially in patients with renal impairment Rarely,
papillary necrosis or interstitial fibrosis associated with NSAIDs may lead to renal failure.
 Hepatic damage, alveolitis, pulmonary eosinophilia, pancreatitis, eye changes, Stevens-Johnson
syndrome and toxic epidermal necrolysis are other rare side-effects. Induction of or exacerbation
of colitis has been reported. Aseptic meningitis has been reported rarely with NSAIDs; patients
with connective-tissue disorders such as systemic lupus erythematosus may be especially
susceptible.

34) D

Indications: Perforation, Haemorrhage, pyloric stenosis (gastric outlet obs), malignant transformation,
failure of medical mx.

35) A

Mallory–Weiss syndrome: A partial-thickness tear. Usually occurs in the right


posterolateral wall of the distal esophagus and results in bleeding that generally
resolves spontaneously. Due to forceful vomiting.

41
36) D

Population Initial Age Recommended Screening Test


Average risk 50 years Annual FOBT or
Flexible sigmoidoscopy every 5 years or
Annual FOBT and flexible sigmoidoscopy
every 5 years or
Air contrast barium enema every 5 years
or
Colonoscopy every 10 years
Adenomatous 50 years Colonoscopy at first detection; then
polyps colonoscopy in 3 years
If no further polyps, colonoscopy every 5
yeaars
If polyps, colonoscopy every 3 years
Annual colonoscopy for >5 adenomas
Colorectal cancer At diagnosis Pretreatment colonoscopy; then at 12
months after curative resection; then
colonoscopy after 3 years; then
colonoscopy every 5 years, if no new
lesions
Ulcerative colitis At diagnosis; then after Colonoscopy with multiple biopsies every
Crohn's colitis 8 years for pancolitis, 1–2 years
after 15 years for left-
sided colitis
FAP 10–12 years Annual flexible sigmoidoscopy
Upper endoscopy every 1–3 years after
polyps appear
Attenuated FAP 20 years Annual flexible sigmoidoscopy
Upper endoscopy every 1–3 years after
polyps appear
HNPCC 20–25 years Colonoscopy every 1–2 years
Endometrial aspiration biopsy every 1–2
years
Familial colorectal 40 years or 10 years Colonoscopy every 5 years
cancer 1st degree before the age of the Increase frequency if multiple family
relative youngest affected members are affected, especially before 50
relative years

42
37) C- Candida dysphagia

38) C

Vitiligo is an acquired, slowly progressive depigmenting condition in small or large areas of the skin due
to the disappearance of previously active melanocytes. White patch, itch in sunlight. Associated with
autoimmune conditions.

· Addison disease
· Alopecia areata
· Chronic mucocutaneous candidiasis
· Diabetes mellitus (insulin dependent)
· Hypoparathyroidism
· Melanoma
· Pernicious anemia
· Polyglandular autoimmune syndrome
· Thyroid disorders (hyperthyroidism and hypothyroidism), 30% of patients
· Uveitis

39) C

Cranial nerve V- Sensory: corneal reflex lost first. Motor- jaw deviates to side of lesion.

40) D

Cavernous venous sinus; thrombosis, ICA aneurysm CN 3, 4, V1, 6.

Berry aneurysm compresses CN 3, 4, 6

41)D

Likely MultipleMyeloma

MULTIPLE MYELOMA
PATHOPHYSIOLOGY
Malignant disease of plasma cells that is characterized by:
_ Presence of monoclonal immunoglobulin or light chains in the serum
and urine
_ Bone destruction (lytic lesions) because myeloma cells produce osteoclastic
activating factor
EPIDEMIOLOGY
_ Median age at diagnosis 68
_ Twice as common in blacks than whites
SIGNS AND SYMPTOMS
_ Hypercalcemia
_ Pathologic fractures/lytic bone lesions
_ Renal failure

43
_ Anemia
DIAGNOSIS
_ Radiography: Lytic lesions on x-ray
_ Electrophoresis: Monoclonal elevation of one immunoglobulin
_ Urinalysis: Free kappa and lambda light chains (Bence Jones proteins)
_ Bone marrow biopsy: 10 to 20% plasma cells (normal is < 5%)
_ Peripheral smear: Rouleaux formation
TREATMENT
There is no cure.
_ Chemotherapy:
_ Alkylating agent (melphalan or cyclophosphamide)
_ Thalidomide (antiangiogenesis agent)
_ Prednisone
_ Calcitonin, bisphosphonates for high calcium
_ Bone marrow transplant is effective.
42) A

Progressive dyspnea, diffuse crackles, clubbing and restrictive lung function test pulmonary fibrosis.

Vs obstructive pattern in COPD (barrel chest) and emphysema.

Question 43- --------------------------------------------answer A


Gilbert’s syndrome- an inherited metabolic disorder. Common cause of unconjugated
hyperbilirubinaemia, due to decreased bilirubin UDP-glucoronysl- transferase activity. Prevalence 1-2%.
Onset shortly after birth but may be unnoticed for many years. Jaundice occurs during intercurrent
illness and bilirubin rises on fasting. It’s a benign condition.

Sickle cell trait does not cause haemolysis. It is usually asymptomatic. If symptoms do occur, it
occurs in situations of hypoxaemia and dehydration eg during strenuous exercise or high
altitudes, not a simple URTI. A summary of the risks associated with sickle cell trait is as
follows.

1. Splenic infarction at high altitude, with exercise, or with hypoxemia


2. Isothenuria with loss of maximal renal concentrating ability

Hematuria secondary to renal papillary necrosis

3. Fatal exertional heat illness with exercise


4. Sudden idiopathic death with exercise
5. Glaucoma or recurrent hyphema following a first episode of hyphema
6. Bacteruria in women
7. Bacteruria or pyelonephritis associated with pregnancy
8. Renal medullary carcinoma in young people (ages 11 to 39 years)
44
9. Early onset of end stage renal disease from autosomal dominant polycystic kidney
disease

Acute hepatitis- transaminases would be markedly elevated

Acute cholecystitis- bilirubin would be conjugated and will be present in urine.

Question 44-------------------------------------- Answer D


This is anorexia nervosa- binge eating/purging type. See page 226 kaplan and saddock. Patient
is underweight and has amenorrhoea. Parotid swelling indicates recurrent vomiting hence binge
eating type. In bulimia nervosa, patient tends to be overweight and should not have
amenorrhoea. Remember that in the diagnosis of bulimia nervosa, the binge eating should not
occur exclusively during periods of anorexia nervosa. Page 228.

Question 45---------------------answer C
Paget’s disease is chacterised by excess osteoclastic bone resorption followed by disordered
osteoblastic activity, leading to abundant new bone formation which is structurally abnormal
and weak. Gene on chrosome 18q osteoclasts contain viral inclusion bodies, suggesting a

possible slow viral etiology. Incidence increases with age. Rare under 40. Affects up to 10 % of
adults by age 90. Clinical features: asymptomatic (most cases) bone pain ( pelvis, lumbrosacral
spine, femur, skull, tibia)deformities eg. Enlargement of skull, bowing of tibia. Complications-
nerve compression (deafness, paresis) fractures, cardiac failure, osteogenic sarcoma.
Investigations: elevated ALP > 1000 U/L with normal calcium and phosphate. X-rays –mixed
osteolytic and osteosclerotic. Treatment- bisphosphonates.

Osteomalacia- ALP increased but calcium and phosphate decreased.

Multiple myeloma- normal ALP elevated calcium. Most present with bone pain

Cirrhosis- mildly elevated ALP as well as transaminases. Serum albumin will be low. Would not
be asymptomatic

Hyperparathyroidism- calcium will be high and phosphate will be low.

Question 46--------------------------------------answer A
Lesions most commonly found in the anterior mediastinum are thymomas, lymphomas,
germ cell tumors, congenital cysts, intrathoracic thyroid tissue, and parathyroid lesions.

45
Thymomas represent 20 percent of all mediastinal neoplasms in adults; they are the most
common anterior mediastinal primary neoplasm in adults, but are rarely seen in children.
They occur with equal frequency in males and females, and commonly present between the
ages of 30 and 50.

Question 47--------------------Answer E
The lenticulostriate artery , a very thin walled branch of the middle cerebral artery usually
involved in hemmhoragic strokes. Hemmhorage involves important descending branches of the
internal capsule and produces hemiplegia on the opposite side. The patient immediately loses
consciousness and paralysis is obvious when consciousness is regained.

Areas supplied by the anterior cerebral artery include

1. The medial surface of the frontal lobe by the medial orbito-frontal artery, and parietal
lobes
2. The anterior four- fifths of the corpus callosum
3. Approximately 1 inch of the lateral surface of frontal and parietal lobe next to the medial
longitudinal fissure
4. Anterior portions of the basal ganglia and internal capsule
5. Olfactory bulb and tract

Occlusion of the anterior cerebral artery may result in the following defects:[citation needed]

1. If stroke occurs prior to the anterior communicating artery it is usually well tolerated
secondary to collateral circulation
2. Paralysis or weakness of the contralateral foot and leg
3. Sensory loss in the contralateral foot and leg
4. Left sided strokes may develop transcortical motor aphasia
5. Gait apraxia
6. Urinary incontinence which usually occurs with bilateral damage in the acute phase

Areas supplied by the middle cerebral artery include:

 The bulk of the lateral surface of the hemisphere; except for the superior inch of the
frontal and parietal lobe (anterior cerebral artery), and the inferior part of the temporal
lobe.

 Superior division supplies lateroinferior frontal lobe (location of Broca's area i.e.
language expression)

46
 Inferior division supplies lateral temporal lobe (location of Wernicke's area i.e. language
comprehension)
 Deep branches supply the basal ganglia as well as the internal capsule

Occlusion of the middle cerebral artery results in Middle cerebral artery syndrome, potentially
showing the following defects:

1. Paralysis (-plegia) or weakness (-paresis) of the contralateral face and arm (faciobrachial)
2. Sensory loss of the contralateral face and arm.
3. Damage to the dominant hemisphere (usually the left hemisphere) results in aphasia i.e.
Broca's or Wernicke's
4. Damage to the non-dominant hemisphere (usually the right hemisphere) results in
contralateral neglect syndrome
5. Large MCA infarcts often have deviation conjugée, a gaze preference towards the side of
the lesion, especially during the acute period. Contralateral homonymous hemianopsia is
often prese

The posterior cerebral artery supplies oxygenated blood to the posterior aspect of the brain (occipital
lobe)

 Contralateral loss of pain and temperature sensations.


 Visual field defects (contralateral hemianopia with macular sparing).
 Prosopagnosia with bilateral obstruction of the lingual and fusiform gyri.
 Superior Alternating Syndrome (Weber's syndrome)
 Contralateral deficits of facial nerve (only lower face, upper face receives bilateral input),
vagus nerve and hypoglossal nerve
 Ipsilateral deficit of occulomotor nerve
 Horner's Syndrome

Question 48---------------------------------------Answer D
This was a difficult question and im not sure if im correct on this one. A stomping gait indicates
sensory ataxia. Spinal cord compression as well as subacute combined degeneration of the cord
can both cause a sensory ataxia but if there was spinal cord compression, the patient would
have other symptoms, especially if bilaterally.

Question 49----------------------------------------AnswerB
47
This patient has an expressive dysphasia which is a lesion in the broca’s motor speech area
which is located in the dominant inferolateral frontal lobe. The patient comprehends speech
well. hence, wernikie’s area is intact. If wernikie’s area was affected, the answer would have
been a.

Question 50 ------------------------Answer D???


This was a tricky question. Two important points. The mother knows the strength of the tablets
and the amount of tabs taken and secondly, the patient presented within half hour. The most
important thing one must do in paracetamol poisoning is to take blood paracetomal levels
because all treatment depends on the blood levels. However, this is done only at or after 4hrs so
I wasn’t sure if this was the correct answer. N-acetylcysteine is only given by IV infusion, not
orally otherwise this would have been the correct answer.

51. A 6 year-old girl is brought to hospital because of numbness of the toes and a history of unsteady gait
for two days. On examination, she is found to have symmetric weakness of the lower limbs there is
difficulty eliciting the deep tendon reflex. The examination of the upper limb is normal and there are no
obvious sensory abnormalities. The remainder of the examination is normal.

Of the following, the most likely diagnosis is:

A. acute cerebellar ataxia


B. acute transverse myelitis
C. cerebrovascular accident
D. Guillian Barre syndrome
E. Poliomyelitis

Answer: D

The typical patient with Guillain-Barré syndrome (GBS), likely AIDP, presents 2-4 weeks
following a relatively benign respiratory or gastrointestinal illness with complaints of finger
dysesthesias and proximal muscle weakness of the lower extremities. The weakness may
progress over hours to days to involve the arms, truncal muscles, cranial nerves, and muscles of
respiration. The illness progresses from days to weeks. The mean time to the clinical function
nadir is 12 days, with 98% of patients reaching a nadir by 4 weeks. A plateau phase of persistent,
unchanging symptoms then ensues, followed days later by gradual symptom improvement. The
mean time to improvement and clinical recovery are 28 and 200 days, respectively.

Symmetric limb weakness typically begins as proximal lower extremity weakness and ascends
to involve the upper extremities, truncal muscles, and head. Inability to stand or walk despite
reasonable strength, especially when ophthalmoparesis or impaired proprioception is present.
Respiratory muscle weakness with shortness of breath may be present.

Paresthesias generally begin in the toes and fingertips, progressing upward but generally not
extending beyond the wrists or ankles. Pain is most severe in the shoulder girdle, back, buttocks,

48
and thighs and may occur with even the slightest movements. Loss of vibration, proprioception,
touch, and pain distally may be present.

52. Of the following, the most likely recognized side effect of sodium valpoate is:

A. diplopia

B.weight gain

C. polyuria

D. gingivitis

E. permanent hair loss

Answer: B

Sodium valproate (INN) or valproate sodium (USAN) is the sodium salt of valproic acid and is an
anticonvulsant used in the treatment of epilepsy and bipolar disorder, as well as other psychiatric
conditions requiring the administration of a mood stabilizer.

The most severe side effect is a ten times higher-than-average incidence rate of serious,
irreversible birth defects (teratogenic) such as births of brainless babies (anencephaly). Risk of
birth defects such as spina bifida has been demonstrated among populations of female patients
who took the medicine in childbearing age.

Otherwise, people who take this drug can experience a variety of side effects, some of which
may require immediate medical attention.

Mild side effects include:

 dizziness
 decreased coordination
 misbalance

Especially dangerous side effects include:

 vomiting
 loss of appetite
 fever
 dark urine

The above side effects suggest a possibility of liver damage. People taking this drug should also
call their doctor if they experience other serious side effects. Some serious side effects include:

49
 unusual bleeding (especially in the urine)
 hallucinations
 drowsiness

Some people also experience:

 constipation
 diarrhea

1% to 10% of people report:

 weight gain
 increased appetite
 abnormal dreams

53. A midwife calls you to see a newborn who has a small penis with an opening at its base and
no palpable testes. The child is otherwise well.

Of the following the most appropriate course of action is

A. reassure the mother that she has a boy


B. reassure the father that he has a girl
C. make a diagnosis of hermaphrodism
D. make a diagnosis of ambiguous genitalia
E. discharge the baby to the endocrine clinic and review in a month

Answer: E

This patient has hypospadias. It is a birth defect of the urethra in the male that involves an
abnormally placed urinary meatus (the opening, or male external urethral orifice). Instead of
opening at the tip of the glans of the penis, a hypospadic urethra opens anywhere along a line
(the urethral groove) running from the tip along the underside (ventral aspect) of the shaft to the
junction of the penis and scrotum or perineum. A distal hypospadias may be suspected even in an
uncircumcised boy from an abnormally formed foreskin and downward tilt of the glans.

The urethral meatus opens on the glans penis in about 50–75% of cases; these are categorized as
first degree hypospadias. Second degree (when the urethra opens on the shaft), and third degree
(when the urethra opens on the perineum) occur in up to 20 and 30% of cases respectively. The
more severe degrees are more likely to be associated with chordee, in which the phallus is
incompletely separated from the perineum or is still tethered downwards by connective tissue, or
with undescended testes (cryptorchidism).

50
First degree hypospadias are primarily a cosmetic defect and have little effect on function except
for direction of the urinary stream. If uncorrected, a second or third degree hypospadias can
make male urination messy, necessitate that it be performed sitting, impair delivery of semen into
the vagina (possibly creating problems with fertility), or interfere with erections. In developed
countries, most hypospadias are surgically repaired in infancy. Surgical repair of first and second
degree hypospadias is nearly always successful in one procedure, usually performed in the first
year of life by a pediatric urologist or a plastic surgeon.

When the hypospadias is third degree, or there are associated birth defects such as chordee or
cryptorchidism, the best management can be a more complicated decision. A karyotype and
endocrine evaluation should be performed to detect intersex conditions or hormone deficiencies.
If the penis is small, testosterone or human chorionic gonadotropin (hCG) injections may be
given to enlarge it prior to surgery.

54. A mother brings her four-year old son to the clinic because she is concerned that he has a
body odour and increasing amounts of acne. She also notes that he is not doing well at school
and is taller than his peers.

Of the following the most likely parameter to detect abnormal pathology is:

A. abnormal body mass index

B. abnormal height velocity

C. abnormal weight velocity

D. normal height velocity

E .normal weight velocity

Answer: B

Honestly did not know the answer here!

55. A three-day infant was noted to have seizures. He was born at term, had normal APGAR
scores at birth and weighed 2kg.

In the initial management of this neonate, the most appropriate anti-epileptic drug is:

A carbamazepine

B. diazepam

C. phenobarbitone

D. phenytoin

51
E. valpoic acid

Answer: C

Phenobarbitone is the drug of choice in the management of neonatal seizures. The dose is
20mg/kg/IV over 20 minutes (not faster than 10mg/kg/min). If seizures persist after this loading
dose, a repeat dose of Phenobarbitone 10mg/kg may be used every 20-30 minutes till a total dose
of 40mg/kg has been given. The maintenance dose is 3-5mg/kg/day in 1-2 divided doses, started
12 hours after the loading dose.

56. Of the following the single best advantage of breast milk compared with cow’s milk is:

A. higher iron concentration

B. higher caloric content per ounce

C. higher Vitamin D concentration

D. higher protein content per ounce

E. higher whey: casein ratio

Answer: E

Higher whey: casein ration is what makes breast milk easily digestible..

57. Which one of the following conditions is inherited as a sex linked single gene defect?

A. duchenne muscular dystrophy

B. Turner syndrome

C. galactosemia

D. achondroplasia

E.cleft lip

Answer: A

Duchenne muscular dystrophy is a sex-linked recessive trait appearing in 20 to 30 per 100000


boys. The disease results from absence of a large protein called dystrophin that is associated with
the muscle fiber plasma membrane.

Clinical features: boys develop an awkward gait and an inability to run properly at 2 to 3 years of
age. Some have an antecedent history of mild slowness in attaining motor milestones, such as

52
walking and climbing stairs. Examination shows firm calf hypertrophy and mild to moderate
proximal leg weakness exhibited by a hyperlordotic, waddling gait and inability to arise from the
ground easily. The child typically arises from a lying position on the floor by using his legs and
body (Gower sign). Arm weakness is evident by 6 years of age, and most boys are confined to a
wheelchair by 12 years of age. Death is caused by pneumonia or congestive cardiac failure
resulting from myocardial involvement.

58. A five-year old child has a patch of alopecia of 2 weeks’ duration. Within the patch are black
dot hairs and scale. Which of the following is the most appropriate treatment?

A. cephalexin orally

B. griseofulvin

C. hydrocortisone

D. miconazole

E. nystatin topically

Answer: B

This patient has tinea capitis. This is infection of the scalp and hair shafts. Microsporum
audouinii previously was responsible for most cases, but Trichophyton tonsurans now causes
more than 90% of cases. Most cases occur in children, with higher incidence among African
American and Hispanic children. Black dot tinea presents with discrete scaly areas of patchy
alopecia with short, broken hairs at the scalp surface, resulting from brittle hairs from intrapilar
sporulation. Other T tonsuras infections present with only mild scalp erythema and fine scaling
that may be itchy or asymptomatic and is easily confused with seborrheic dermatitis( dandruff)
and atopic dermatitis. Diffuse hair loss may complicate long standing infection.

The diagnosis is established by clinical appearance and may be confirmed by KOH examination
of a hair plucked from the scalp or by fungal culture. Tinea capitis is treated with oral
griseofulvin for 6 to 8 weeks or terbinafine or itracoonazole for 2 to 4 weeks. Adjunctive
useof selenium sulfide or ketoconazole shampoo may decrease spore shedding and reduce
transmission.a short course of oral prednisolone may hasten resolution and reduce the risk of
scarring.

59. Ans:A. Excision. Mantoux positive if 10mm induration.

60. Acute pyelonephritis normal discriminatory zone is 105 but if highly suggestive features is
pyelonephritis.

61. D chronic pulmonary interstitial disease associated with a lymphocytic infiltrate of lung. The
disease is lymphocytic interstitial pneumonia and it is rare.

53
Serum Igs usually normal may be slightly elevated. No increase in ability to respond to tetanus
toxoid. Diagnosis can be confirmed by 2-3months with PCR RNA more accurate than DNA
about 100% sensitive and specific at 3months. 18months is time taken for maternal antibody to
disappear from serum

62. Ans: E

Diagnostic Criteria(Modified Jones Criteria)

Revised Jones Criteria for Acute Rheumatic Fever (ARF)

A firm diagnosis requires that two major or one major and two minor criteria are satisfied,
in addition to evidence of recent streptococcal infection.

Major Criteria

1. Carditis: All layers of cardiac tissue are affected (pericardium, epicardium,


myocardium, endocardium) The patient may have a new or changing murmur, with
mitral regurgitation being the most common followed by aortic insufficiency.
2. Polyarthritis: Migrating arthritis that typically affects the knees, ankles, elbows and
wrists. The joints are very painful and symptoms are very responsive to anti-
inflammatory medicines.

3. Chorea: Also known as Syndenham´s chorea, or "St. Vitus´ dance". There are
abrupt, purposeless movements. This may be the only manifestation of ARF and is its
presence is diagnostic. May also include emotional disturbances and inappropriate
behavior.

4. Erythema marginatum: A non-pruritic rash that commonly affects the trunk and
proximal extremities, but spares the face. The rash typically migrates from central
areas to periphery, and has well-defined borders.

5. Subcutaneous nodules: Usually located over bones or tendons, these nodules are
painless and firm.

Minor Criteria:

1. Fever
2. Arthralgia

3. Previous rheumatic fever or rheumatic heart disease

4. Acute phase reactants: Leukocytosis, elevated eritrosedimentation rate (ESR) and C-


reactive protein (CRP)

5. Prolonged P-R interval on electrocardiogram (ECG)

Evidence of preceding streptococcal infection: Any one of the following is considered


adequate evidence of infection:

54
 Increased antistreptolysin O or other streptococcal antibodies
 Positive throat culture for Group A beta-hemolytic streptococci

 Positive rapid direct Group A strep carbohydrate antigen test

 Recent scarlet fever.

63. D. Transposition of the great vessels. Usually cyanosis resistant to oxygen, occurs at 1-2days
when ductus closes . Echocardiogram confirms the anatomy. Function better if other associated
defects eg VSD. Mx: PGE1 to dilate PDA, O2 , bicarb, diuretics. Surgical: short term:ballon
atrial septostomy to allow mixing while awaiting arterial switch procedure.

Hypoplastic left heart syndrome(due to failure development mitral/aortic valve/ aortic arch) cant
pump enough blood has similar presentation but less common: 1% vs 5%.

Tetralogy of Fallot:

1. VSD
2. AORTA OVERRIDES VSD
3. PULMONARY STENOSIS
4. BOOT SHAPED HEART

Characterised by hypoxic spells “tet spells” that respond to oxygen and knee chest position

Congestive heart failure in infants is usually characterized by: failure to thrive, increased work
breathing, poor feeding and excessive sweating especially during feeding.

64. C: Bronchiolitis: typical age

Atypical pneumonia: 12

Bronchopneumonia: 2-5

Viral myocarditis rare: presents with flu like illness then heart failure, chest pain, dilated
cardiomyopathy

65. B immune mediated. This sounds like Kawasaki disease

Fever for five days or more plus 4 of:


1. Bilateral conjunctival injection without exudate
2. Polymorphous exanthem
3. Changes in lips and mouth:
 Reddened, dry, or cracked lips
 Strawberry tongue
 Diffuse redness of oral or pharyngeal mucosa
4. Changes in extremities:
 Reddening of palms or soles
 Indurative oedema of hands or feet

55
 Desquamationof skin of hands, feet, and groin (in convalescence)
5. Cervical lymphadenopathy:
 More than 15 mm in diameter, usually unilateral, single, non-purulent, and painful
Stevens Johnson syndrome usually associated with drug exposure and a shorter course, and
erosion of mucosa rather than just hyperemia. Target lesions which coalesce to large patches face
and trunk form frank bullae.

Scarlet fever: circumoral pallor, strawberry tongue(red or white)

66. D. AVSD( endocardial cushion defect) assoc with down

67. Bullous impetigo Staph aureus most common cause

67. E

Skin infections are the most common type of disease produced by Staphylococcus. Staph
infections of the skin can progress to impetigo (a crusting of the skin) or cellulitis (inflammation
of the connective tissue under the skin, leading to swelling and redness of the area). In rare cases,
a serious complication known as scalded skin syndrome can develop. Scalded skin
syndrome: A potentially serious side effect of infection with the Staph (Staphylococcus) bacteria
that produces a specific protein which loosens the "cement" holding the various layers of the skin
together. This allows blister formation and sloughing of the top layer of skin. If it occurs over
large body regions it can be deadly (just like a large surface area of the body having been
burned). It is necessary to treat scalded skin syndrome with intravenous antibiotics and to protect
the skin from allowing dehydration to occur if large areas peel off. The disease occurs
predominantly in children under 5 years of age. It is known formally as Staphylococcal scalded
skin syndrome and as Ritter disease.

68. D

Diaper rashes are common in babies between 4 and 15 months old. They may be noticed more
when babies begin to eat solid foods.Diaper rashes caused by infection with a yeast or fungus
called Candida are very common in children. Candida is found everywhere in the environment. It
grows best in warm, moist places, such as under a diaper. A yeast-related diaper rash is more
likely to occur in babies who:

 Are not kept clean and dry


 Are taking antibiotics, or whose mothers are taking antibiotics while breast feeding
 Have more frequent stools

Other causes of diaper rashes include:

 Acids in the stool (seen more often when the child has diarrhea)
 Ammonia (produced when bacteria break down urine)

56
 Diapers that are too tight or rub the skin
 Reactions to soaps and other products used to clean cloth diapers
 Too much moisture

The best treatment for a diaper rash is to keep the diaper area clean and dry. This will also help
prevent new diaper rashes. Topical antifungal skin creams and ointments will clear up infections
caused by yeast. Nystatin, miconazole, clotrimazole, and ketaconazole are common ones.
69. D
Eczema is a chronic skin disorder that involves scaly and itchy rashes. Atopic eczema is the most
common type. Atopic eczema is due to a hypersensitivity reaction (similar to an allergy) in the
skin, which leads to long-term inflammation of the skin.
Part of the treatment in eczema is to keep the skin moist (called lubricating or moisturizing the
skin). Use ointments (such as petroleum jelly), creams, or lotions 2 - 3 times a day. Moisturizers
should be free of alcohol, scents, dyes, fragrances, or other chemicals. A humidifier in the home
will also help.
70. A –pg 31 Illustrated textbook of paediatrics , 3rd Ed. A 3 yr old child should be able to speak
in 3 word sentences, one who stutters has a language / medical problem
71. A
Epiglottitis is swelling of the epiglottis, which may block the windpipe and lead to breathing
problems. Swelling of the epiglottis is usually caused by the bacteria Haemophilus influenzae
(H. influenzae), although it may also be caused by other bacteria or viruses that lead to upper
respiratory infections.
Epiglottitis is now very uncommon, but it was most common in children ages 2 - 6. Rarely,
epiglottitis can occur in adults, and it may be easily overlooked in such patients.
Epiglottitis begins with a high fever and sore throat. Other symptoms may include:
 Abnormal breathing sounds (stridor)
 Chills, shaking
 Cyanosis (blue skin coloring)
 Drooling
 Difficulty breathing (patient may need to sit upright and lean slightly forward to
breathe)
 Difficulty swallowing
 Voice changes (hoarseness)
Epiglottitis is a medical emergency. Seek immediate medical help. Do not use a tongue depressor
(tongue blade) to try to examine the throat at home, as this may make the condition worse.

57
The health care provider will examine the voice box (larynx) using either a small mirror held
against the back of the throat or a viewing tube called a laryngoscope. (See: laryngoscopy) The
exam may show a swollen and red epiglottis.
Tests used to diagnose epiglottitis may include:
 Blood culture or throat culture -- may show H. influenzae or other bacteria
 Complete blood count (CBC) -- may show a high number of white blood cells
 Neck x-rays -- may show a swollen epiglottis
Epiglottitis may require urgent tracheal intubation to protect the airway, though this is not always
the case. In some cases, Epiglottitis requires the use of antibiotics while a patient is experiencing
the benefits of a breathing tube. In more serious cases, tracheal intubation is necessary. In such
cases, it is not advised to immediately head in the direction of intubation because the inflammed
epiglottis is very sensitive and if you irritate the epiglottis with the laryngoscope you can cause
the epiglottis to close off completely forcing the use of a surgical airway (cricothyrotomy). Most
children can be managed by letting the child be in a position of comfort, keep the lights down
low and keep the child calm. Intubation is a good thing to have in the back of your mind and it
may become necessary if the child starts to rapidly decompensate and show signs of impending
respiratory arrest (Decreased work of breathing with abnormal skin signs) given in the initial
stages to reduce symptoms, but this will not treat the underlying cause. It should also be noted
that if stridorbecomes quieter, obstruction is likely to follow, and thus intubation should be
expedited even further.
72. C
For a patient in whom PEA is suspected, the American Heart Association - Advanced Cardiac
Life Support (AHA-ACLS) guidelines protocol (revised in 2010[10, 11] ) recommends the
following:[12]
 Initiate CPR.
 Place an intravenous line.
 Intubate the patient.
 Correct hypoxia by administering 100% oxygen.
Once these basic measures are in place, reversible causes should be sought and corrected, which
include the following:
 Hypovolemia
 Hypoxia
 Acidosis
 Hypokalemia/hyperkalemia
 Hypoglycemia
 Hypothermia
 Toxins (eg, tricyclic antidepressants, digoxin, calcium channel blocker, beta-blockers)
 Cardiac tamponade
 Tension pneumothorax
 Massive pulmonary embolus
 Acute myocardial infarction
The clinical scenario usually provides useful information. Some examples include the following:
58
 In a previously intubated patient, tension pneumothorax and auto-PEEP are more likely to
occur.
 In a patient on dialysis, consider hyperkalemia.
 In a patient with prior myocardial infarction or CHF, myocardial dysfunction is likely.
 A core temperature should always be obtained if the patient is thought to have
hypothermia.
 In patients diagnosed with hypothermia, resuscitative efforts should be continued at least
until the patient is rewarmed because patient survival is possible even after prolonged
resuscitation.[13]
Other components of the evaluation include the following:
 Measure QRS duration since it has prognostic significance. Patients with QRS duration
of less than 0.2 second are more likely to recover, and high-dose epinephrine may be
administered. Acute rightward axis shifts can suggest possible pulmonary embolus.
 Invasive monitoring (eg, arterial line) may be placed if it does not cause a delay in
delivering standard ACLS care.
 Echocardiography, if available, may assist with identifying the presence of cardiac
contractions (pseudo-PEA). Patients with pseudo-PEA may have a rapidly reversible cause (eg,
auto-PEEP, hypovolemia). Echocardiography also is invaluable in identifying cardiac
tamponade,[6]right ventricular enlargement, pulmonary hypertension suggestive of pulmonary
emboli, myocardial dysfunction, cardiorrhexis, or ventricular septal rupture.
 In refractory cases, if the patient has suffered chest trauma, a thoracotomy may be
performed, provided adequate expertise is available.
Once reversible causes are identified, they should be corrected immediately. This process
involves needle decompression of pneumothorax, pericardiocentesis for tamponade, volume
infusion, correction of body temperature, and administration of thrombolytics or surgical
embolectomy for pulmonary embolus.
Resuscitative pharmacology includes epinephrine, vasopressin, and atropine. Epinephrine should
be administered in 1 mg doses IV/IO q3-5min during PEA arrest. Higher doses of epinephrine
have been studied and show no improvement in survival or neurologic outcomes in most
patients. Special populations of patients, such as those who have overdosed on beta-blockers and
calcium channel blockers may benefit from higher dose epinephrine. Vasopressin 40 U IV/IO
may replace either the first or second dose of epinephrine in patients with pulseless electrical
activity.[14, 15]
If the underlying rhythm is bradycardia (ie, heart rate < 60 bpm) associated with hypotension,
then atropine (1 mg IV q3-5min, up to 3 doses) should be administered. This is considered the
total vagolytic dose, beyond which no further benefit will occur. Note that atropine may cause
pupillary dilation, and this sign then cannot be used to assess neurologic function.
Sodium bicarbonate may be administered only in patients with severe systemic acidosis,
hyperkalemia, or a tricyclic antidepressant overdose. The dose is 1 mEq/kg. Routine
administration is discouraged because it worsens intracellular and intracerebral acidosis and does
not appear to alter the mortality rate.
Prompt initiation of a cardiopulmonary bypass may have a role in carefully selected patients.
This maneuver requires availability of expertise and support services. Patient selection is

59
paramount because it should be used only in patients who have an easily reversible etiology of
cardiac dysfunction. In an animal model, initiation of prompt cardiopulmonary bypass resulted in
a higher rate of success in returning circulation than administration of high- or standard-dose
epinephrine. Cardiac pacing can result in electrical capture but does not necessarily increase the
incidence of mechanical contractions; hence, this procedure is not recommended.
Near pulseless electrical activity, or a profound low output state, may also be addressed with
different means of circulatory assist (eg, intra-aortic balloon pump, extracorporeal membrane
oxygenation, cardiopulmonary bypass, ventricular assist device)
73. B

74. B
Reasons for not giving polio vaccine:
 Acute illness with a raised temperature. Postpone until better.
 Diarrhoea and or vomiting. Postpone until better.
 Treatment with high doses of steroids or drugs that suppress the immune system, also
some forms of radiation therapy.

60
 Conditions which suppress the immune system such as leukaemia and lymphomas.
 First four months of pregnancy, although no evidence exists of harm coming to a baby
born after such administration of vaccine. Exceptions may rarely be made, if the
circumstances are compelling (eg need to travel to an area where polio is rife).
 If you are allergic to certain antibiotics (penicillin, neomycin, polymyxin, or
streptomycin) you should inform the doctor or nurse before you receive any medication.
Unless the sensitivity is extreme to any of these, it need not contraindicate using the
vaccine.
In the event of being unable to receive oral polio vaccine, because of having an immune system
which is suppressed (the individual is said to be immunosuppressed or immunodepleted), by
drugs or disease, there is an alternative vaccine which is not live (Inactivated Polio Vaccine). It is
also used for members of the household of such individuals, if they require polio vaccination.
People with HIV can receive oral polio vaccine, but they may excrete the attenuated virus in
their faeces for longer than other people.

75. A

A maculopapular rash is a type of rash characterized by a flat, red area on the skin that is
covered with small confluent bumps. The term "maculopapular" is a compound: macules are
small, flat discolored spots on the surface of the skin; and papules are small, raised bumps. It is
also described as erythematous, or red. This type of rash is common in several diseases and
medical conditions, including scarlet fever, measles, secondary syphilis, erythrovirus (parvovirus
B19), and heat rash. It is also a common manifestation of a skin reaction to the
antibiotic amoxicillin or chemotherapy drugs. Cutaneous infiltration of leukemic cells may also
have this appearance. Maculopapular rash is seen in graft-versus-host disease (GVHD)
developed after a blood transfusion, which can be seen within one week or several weeks after
the blood transfusion. In the case of GVHD, the maculopapular rash may progress to a condition
similar totoxic epidermal necrolysis.

The primary symptom of rubella virus infection is the appearance of a rash (exanthem) on the
face which spreads to the trunk and limbs and usually fades after three days (that is why it is
often referred to as three-day measles). The facial rash usually clears as it spreads to other parts
of the body. Other symptoms include low grade fever, swollen glands (sub occipital & posterior
cervical lymphadenopathy), joint pains, headache and conjunctivitis.
The rash associated with Rocky Mountain spotted fever usually starts around 4 days into the
illness. It looks like small, red, flat spots starting most often on the ankles and wrists, and then
moving to the palms, soles, and trunk. As the rash progresses, it becomes bumpier.

Exanthema subitum (meaning sudden rash), also referred to as roseola infantum (or rose rash of
infants), sixth disease (as the sixth rash-causing childhood disease) and (confusingly) baby
61
measles, orthree-day fever, is a disease of children, generally under two years old,although it has
been known to occur in eighteen year olds, whose manifestations are usually limited to a
transient rash ("exanthem") that occurs following a fever of about three day's duration. It is
frequently called roseola, although this term could be applied to any rose-colored rash. Typically
the disease affects a child between six months and two years of age, and begins with a sudden
highfever (39–40 °C). After a few days the fever subsides, and just as the child appears to be
recovering, a red rash appears. This usually begins on the trunk, spreading to the legs and neck.
The rash is not itchy and may last 1 to 2 days.

Scarlet fever is a disease caused by exotoxin released by Streptococcus pyogenes. Once a major
cause of death, it is now effectively treated with antibiotics.

It is characterized by:

 Sore throat
 Fever

 Bright red tongue with a "strawberry" appearance

 Characteristic rash, which:

 is fine, red, and rough-textured; it blanches upon pressure.

 appears 12–48 hours after the fever.

 generally starts on the chest, armpits, and behind the ears.

 spares the face (although some circumoral pallor is characteristic).

 is worse in the skin folds. These Pastia lines (where the rash runs together in the
armpits and groin) appear and can persist after the rash is gone.

 may spread to cover the uvula.

 The rash begins to fade three to four days after onset and desquamation (peeling) begins.
"This phase begins with flakes peeling from the face. Peeling from the palms and
around the fingers occurs about a week later."[3] Peeling also occurs in axilla, groin.

76. b - Colon

77. ans d herpes hominis virus type 6

62
HHV 6 is classically associated with exanthema subitum (roseola infantum) where there is a high fever
with malaise lasting a few days, followed by a general macular rash which appears as the fever wanes.
Primary HHV6 is a common cause of febrile convulsions associated with up to 1/3 of such in the first year
of life.

78. ans d 6 stools per day

How many stools are normal for a breast feeding baby? During the first few days after birth (meconium
phase) up to a total of 5 in a few days. Once milk matures into colostrums, 2-5 bowel movements per
day in the first 6 weeks

79. ans d perform barlow and ortolani maneuvers

early dx by postnatal screening w/ Ortolani or Barlow's Test is essential to detect hip instability or
dislocation;
- strictly interpreted, the Ortolani sign is the palpable sensation of the gliding of the femoral head in
and out of the acetabulum;
- femoral head glides back and forth over a ridge of pathologic hypertrophic acetabular cartilage as
the femoral head enters and leaves the acetabulum;
- others have interpreted the Ortolani sign as the palpable reduction of an infant's dislocated hip
where as the Barlow's Test is used to describe the provoked
dislocation of an unstable hip by gently adducting flexed hip while pushing posteriorly in line of
shaft of femur;
- Ortolani test identifies dislocated hip that can reduced in early weeks of life

80. ans c septal hypertrophy

Cardiomegaly is common (30%), and heart failure occurs in 5–10% of infants of diabetic mothers.
Asymmetric septal hypertrophy may occur and become manifested similar to transient idiopathic
hypertrophic subaortic stenosis. Inotropic agents worsen the obstruction and are contraindicated.
Congenital heart disease is more common in infants of diabetic mothers. Birth trauma is also a common
sequela of fetal macrosomia.

81. ans b complete liver functions tests

Patient is icteric, no fever!

82. ans b abo incompatability

Neonatal jaundice <24hrs of age: hemolytic disorders, rhesus incompatability, ABO


incompatability, G6pd deficiency, sperocytosis, pyruvate kinase deficiency, congenital infection.

63
ABO more common, IgM doesn’t cross placenta, but some type O mothers have IgG anti A
hemolysin (anti B possible too)

83. d streptococcus pneumoniae

Most likely (staph aureus 2nd), according to the OHof paediatrics a patient infected with s.pneumoniae
may hae pleuritic chest pain and abdominal pain!

84) B- A known complication of Otitis media is meningitis or brain abscess. CSF analysis will
aid in the diagnosis and management of the patient.

85) E- The history is characteristic of an Absence Seizure; which are brief staring spelling
typically lasting less than 10 secs without a postictal phase. They are generalized seizures with
an EEG showing 3 Hz spike and wave activity. Remission tend to occur by adolescence and
treatment u usually withdrawn after a seizure free period of four years.

86) B- Cereals are usually started after 6 mo but they are not added to bottles rather a spoon
should be used. Whole milk is not recommended before the age of 12 months. Formula fed
infants tend to gain weight faster due to water retention and different fat composition while
breastfed infants are leaner. The extrusion reflex is a normal response to force the tongue
outward when touched or depressed. This aids in feeding and is present until 4 mo.

87) B- Echopraxia is the involuntary imitation of movement

88) C-
Good prognosis Poor prognosis
Late onset young onset
Obvious precipitating factors No precipitating factors
Acute onset Insidious onset
Mood disorder symptoms Poor premorbid social, sexual hx
Family support No family spport
Positive symptoms Neurological signs and symptoms
Negative symptoms

89) B- Abstract thinking: Proverb interpretation is dependent on education and cultural beliefs.

90) B- Obsessive Compulsive disorder is the repetition of senseless rituals, perceived by the
individual as nonsensical. Obsessive compulsive personality disorder is a rigid obstinate bigot
who is preoccupied with unimportant detail. Hypochondriasis is the magnification of severity or
importance of physical symptoms. Social phobia is the fear of social interaction.

91) C- Delusional disorder in which primary or sole manifestation is a delusion that is fixed and
unshakable. They are non- bizarre delusions involving real life situations but this doesn’t impair
individual’s level of functioning.

92. The most likely pathological changes characteristic of Alzheimer’s disease is:

64
(a) Enlarged ventricles and narrowed sulci

(b) plaques of amyloid : ↑ number of senile plaques

(c) Focal loss of neurons confined to subcortical areas : neuronal loss is selective , and the hippocampus,
amygdale, temporal neocortex and some subcortical nuclei eg. The nucleus basalis of meynert are
particularly vulneralble.

(d) increased cholinergic neurons: loss of neurotransmitter choline

(e) neurofibrillary tangles : Accumulation of beta amyloid peptide, a degradation product of amyloid
precursor protein,→ progressive neuronal damage and neurofibrillary tangles

93. A mother who has a child she does not want becomes very protective of the child. This is most likely
an example of the defence mechanism:

The individual deals with emotional conflict or internal or external stressors by

(a) sublimation:

(b) suppression: intentionally avoiding thinking about disturbing problems, wishes, feelings or
experiences

(c) reaction formation: substituting behaviour thoughts , or feelings that are diametrically opposed to
his or her own acceptable thoughts or feelings (this usually occurs in conjunction with their
repression)

(d) rationalization: concealing the true motivations for his or her thoughts , actions or feelings through
the elaboration of reassuring or self-serving but incorrect explanations

(e) projection: falsely attributing to another his or her own unacceptable feelings, thoughts or impulses

94.A new office screening test for haematuria claims to be 95% sensitive and 100 % specific. In using the
test in an average clinic population, the most likely correct response is:

(a) few false positives

(b) many false positives

(c) no false positives

(d) no false negatives

(e) many false negatives

Test result Patient with the condition Patient without the condition
Subjects appear to have the True +ve (a) False +ve (b)
condition
Subjects appear to NOT have the False -ve (c) True –ve (d)

65
condition

Specificity: How reliable is the test –ve in health? d/d+b.. if the test is 100% specific then there should be
no false +ves and d/d= 1 (100%).

Sensitivity: How reliable is the test +ve in health? a/a+c.. if the test is 100% sensitive then there should
be no false –ve’s and a/a = 1 ( 100%)

95. The high prevalence of the Aedes Aegypti mosquito in a community will most likely contribute to
increasing the risk of which of the following conditions?

(a) Malaria

(b) Filariasis

(c) AIDS

(d) yellow fever

(e) Dengue fever: Spread by the Aedes Aegypti mosquitoe horses, animals, people.

Risks for Dengue Haemorrhagic Fever/ Dengue Shock Syndrome

i. Previous infection
ii. > 1 serotype within the blood
iii. > 1 serotype within the community/ geographic area.
iv. Age ( young and the old)

96. A statistical report states that in a certain country, approximately 12 people die from CVAs every year
for every 100,000 persons estimated to be alive in that country in that year. Which of the following
measures can be best obtained from this information?

(a) case fatality rates for stroke

(b) standardized mortality rate for stroke ( number of deaths due to a particular cause per 100,000)

(c) crude mortality rate for stroke

(d) adjusted mortality rate for stroke

(e) risk ratio of death from strokes

66
97. An epidemic investigation of food poisoning arising from a church picnic showed that the median
time for eating potato salad and having symptoms of diarrhea and vomiting was 14 hours. Which of the
following best describes the time interval?

(a) communicable period

(b) pre-pathogenic period

(c) pre-infectious period

(d) incubation period

(e) period of subclinical disease

98. in a study which began in 2006, 3,000 adults were asked about past alcohol consumption. The
occurrence of cases of liver cancer will be determined between 2006 and 2009. Of the following, which
best describes this type of study?

(a) cross- sectional study

(b) concurrent cohort study correct answer http://www.hsrmethods.org/Glossary/Terms/C/Concurrent


%20Cohort%20Study.aspx

(c) retrospective cohort study (the study group consists of subjects exposed the putative causal factor;
and the control group consists of subjects not exposed. The incidence of the disease is compared
between groups over time ) steups

(d) clinical trial

(e) case- control study

99. Of the following, which measure is the most likely to be used as a numerator to calculate prevalence
rate?

Prevalence: The number of new and old existing cases of a disease in a given population

At a specific period of time (period prevalence)

or at a particular moment in time (point prevalence)

# of new and old existing cases x100

Population at risk

(a) total population at risk

(b) Number of new cases

(c) number of new and existing cases

67
(d) number of pre-existing cases

(e) person-years of observation

100. In 8 families surveyed, the numbers of children per family were 3,2,6,2,2,5,5, and 7. Which of the
following combination of numbers best represents the mean, median and mode (in that order) numbers
of children per family?

(a) 4,4,2

(b) 4,3,2

(c) 3,4,2

(d) 4,5,2

(e) 4,4,3

68

Vous aimerez peut-être aussi